You are on page 1of 130

Observe 

high level of confidentiality over this document
This may serve as a test of mastery
Study not based on the answers but based on your learning/understanding before referring to the answers 
provided. 
Know first the concepts before opening this reviewer. 
GOD BLESS! 

STANDARD COSTING REVIEW MATERIALS:

D 1. The type of standard that is intended to represent challenging yet attainable results is:
A. theoretical standard
B. flexible budget standard
C. controllable cost standard
D. normal standard
E. expected actual standard

A 2. Standard costs are used for all of the following except:
A. income determination
B. controlling costs
C. measuring efficiencies
D. forming a basis for price setting
E. establishing budgets

C 3. Of the following variances, the one that is most useful in assessing the performance of the Purchasing 
Department is the:
A. idle capacity variance
B. overhead price variance
C. materials purchase price variance
D. labor rate variance
E. materials price usage variance

B 4. The labor efficiency variance is computed as:
A. the difference between standard and actual rates, multiplied by standard hours
B. the difference between standard and actual hours, multiplied by standard rate
C. the difference between standard and actual rates, multiplied by actual hours
D. the difference between standard and actual hours, multiplied by the difference between standard
and actual rates
E. a percentage of the labor time variance

1
2 Chapter 18

B 5. The method used to assure fairness in the rates paid for each operation performed by an employee is:
A. job costing
B. job rating
C. union contracting
D. the agreed­upon wages at the time of employment
E. labor rate variance analysis

D 6. Materials and labor cost standards are generally based on:
A. expected actual conditions, anticipated prices, and desired efficiency levels
B. theoretical conditions, present price levels, and desired efficiency levels
C. capacity conditions, anticipated prices, and desired efficiency levels
D. normal conditions, present price levels, and desired efficiency levels
E. theoretical conditions, anticipated prices, and theoretically attainable efficiency levels

D 7. The most effective standards are set following a careful study of products and operating conditions by 
the:
A. Accounting Department, central management, and the Industrial Engineering Department
B. central management and the employees whose performance is being evaluated
C. Accounting Department and engineering staff
D. Industrial Engineering Department and the employees whose performance is being evaluated
E. central management and the Industrial Engineering Department

E 8. In analyzing factory overhead variances, the volume variance is the difference between the:
A. actual amount spent for overhead items during the period and the amount applied during the 
period
B. variable efficiency variance and fixed efficiency variance
C. amount shown in the flexible budget and the amount shown in the master budget
D. master budget application rate and the flexible budget application rate, multiplied by actual 
hours worked
E. budget allowance based on standard hours allowed for actual production for the period and the 
amount of applied factory overhead during the period

D 9. The variance resulting from obtaining an output different from the one expected on the basis of input 
is the:
A. mix variance
B. output variance
C. usage variance
D. yield variance
E. efficiency variance

A 10. In its reports to management, a company disclosed the presence of a fixed efficiency variance.  The 
procedure used to analyze variances was the:
A. four­variance method
B. mix and yield variances method
C. two­variance method
D. alternative three­variance method
E. three­variance method
D 11. A purpose of standard costing is to:
A. allocate cost with more accuracy
B. eliminate the need for subjective decisions by management
C. determine the "break­even" production level
D. control costs
E. all of the above

A 12. Which one of the following is true concerning standard costs?
A. If properly used, standards can help motivate employees.
B. Unfavorable variances, material in amount, should be investigated, but large favorable 
variances need not be investigated.
C. Standard costs are difficult to use with a process costing system.
D. Standard costs are estimates of costs attainable only under the most ideal conditions, but rarely 
practicable.
E. All of the above

A 13. When computing variances from standard costs, the difference between actual and standard price 
multiplied by actual quantity yields a:
A. price variance
B. volume variance
C. mix variance
D. yield variance
E. combined price­quantity variance

E 14. A company controls its production costs by comparing its actual monthly production costs with the 
expected levels.  Any significant deviations from expected levels are investigated and evaluated as a 
basis for corrective actions.  The quantitative technique that is most probably being used is:
A. time­series or trend regression analysis
B. correlation analysis
C. differential calculus
D. risk analysis
E. standard cost variance analysis

C 15. What type of direct material variances for price and usage will arise if the actual number of pounds of 
materials used was less than standard pounds allowed but actual cost exceeds standard cost?

Usage Price
A. unfavorable favorable
B. favorable favorable
C. favorable unfavorable
D. unfavorable unfavorable
E. none none

B 16. If a company follows a practice of isolating variances at the earliest time, the appropriate time to 
isolate and recognize a direct materials price variance would be when:
A. the purchase order is originated
B. materials are purchased
C. materials are issued
D. the materials requisition is prepared
E. materials are used in production
A 17. Which of the following would least likely cause an unfavorable materials quantity (usage) variance?
A. labor that possesses skills equal to those required by the standards
B. scheduling of substantial overtime
C. a mix of direct materials that does not conform to plan
D. materials that do not meet specifications
E. machinery that has not been maintained properly

D 18. Information about Sargent Company's direct material costs is as follows:

Standard unit price $3.60
Actual quantity purchased 1,600
Standard quantity allowed for actual production 1,450
Materials purchase price varianceCunfavorable $240

What was the actual purchase price per unit, rounded to the nearest penny?
A. $3.06
B. $3.11
C. $3.45
D. $3.75
E. $3.60

SUPPORTING CALCULATION:

$240 = 1,600 (x ­ $3.60)
1,600 x = $240 + $5,760
x = $3.75

C 19. Using the following symbols, which formula represents the calculation of the labor rate variance?

AH = Actual hours
SH = Standard hours allowed for actual production
AR = Actual rate
SR = Standard rate

A. SR(AH ­ SH)
B. AR(AH ­ SH)
C. AH(AR ­ SR)
D. SH(AR ­ SR)
E. SH(SR ­ AR)

D 20. When a change in the manufacturing process reduces the number of direct labor hours and standards 
are unchanged, the resulting variance will be:
A. an unfavorable labor usage variance
B. an unfavorable labor rate variance
C. a favorable labor rate variance
D. a favorable labor usage variance
E. both (C) and (D) above
B 21. The most probable reason a company would experience a favorable labor rate variance and an 
unfavorable labor efficiency variance is that:
A. the mix of workers assigned to the particular job was heavily weighted toward the use of higher
paid, experienced individuals
B. the mix of workers assigned to the particular job was heavily weighted toward the use of new, 
relatively low­paid, unskilled workers
C. because of the production schedule, workers from other production areas were assigned to assist
in this particular process
D. defective materials caused more labor to be used in order to produce a standard unit
E. the actual price paid for materials that went into production was less than the standard price that
was expected to be paid

C 22. Information on Orman Company's direct labor costs is as follows:

Standard direct labor rate................................................................................................. $3.75
Actual direct labor rate..................................................................................................... $3.50
Standard direct labor hours.............................................................................................. 10,000
Direct labor usage (efficiency) varianceCunfavorable.................................................... $  4,200

What were the actual hours worked, rounded to the nearest hour?
A. 11,914
B. 10,714
C. 11,120
D. 11,200
E. none of the above

SUPPORTING CALCULATION:

$4,200 = $3.75 (x ­ 10,000)
$3.75 x = $4,200 + $37,500
x = 11,120

D 23. Each unit of Product 8in1 requires two direct labor hours. Employee benefit costs are treated as direct 
labor costs. Data on direct labor are as follows:

Number of direct employees............................................................................................ 25
Weekly productive hours per employee........................................................................... 30
Estimated weekly wages per employee............................................................................ $240
Employee benefits (related to weekly wages).................................................................. 25%

The standard direct labor cost per unit of Product 8in1 is:
A. $8.00
B. $10.00
C. $12.00
D. $20.00
E. none of the above

SUPPORTING CALCULATION:
B 24. J. R. Richard Company employs a standard absorption system for product costing.  The standard cost 
of its product is as follows:

Direct materials................................................................................................................ $14.50
Direct labor (2 direct labor hours x $8)............................................................................ 16.00
Manufacturing overhead (2 direct labor hours x $11)......................................................  22.00
Total standard cost........................................................................................................... $52.50

The manufacturing overhead rate is based upon a normal activity level of 600,000 direct labor hours.  
Richard planned to produce 25,000 units each month during the year.  The budgeted annual 
manufacturing overhead is:

Variable............................................................................................................................ $3,600,000
Fixed.................................................................................................................................  3,000,000
.......................................................................................................................................... $6,600,000

During November, Richard produced 26,000 units.  Richard used 53,500 direct labor hours in 
November at a cost of $433,350.  Actual manufacturing overhead for the month was $250,000 fixed 
and $325,000 variable.

The manufacturing overhead controllable variance for November is:
A. $9,000 unfavorable
B. $13,000 unfavorable
C. $9,000 favorable
D. $4,000 favorable
E. none of the above

SUPPORTING CALCULATION:

Actual factory overhead................................................... $ 575,000
Budget allowance:
Variable factory overhead (52,000 x $6).................. $312,000
Budgeted fixed overhead..........................................   250,000         562,000
Controllable variance.......................................................  $       
   13,000 unfavorable
B 25. J. R. Richard Company employs a standard absorption system for product costing.  The standard cost 
of its product is as follows:

Direct materials.............................................................................................................. $14.50
Direct labor (2 direct labor hours x $8)......................................................................... 16.00
Manufacturing overhead (2 direct labor hours x $11)...................................................    22.00
Total standard cost......................................................................................................... $52.50

The manufacturing overhead rate is based upon a normal activity level of 600,000 direct labor hours.  
Richard planned to produce 25,000 units each month during the year.  The budgeted annual 
manufacturing overhead is:

Variable.......................................................................................................................... $3,600,000
Fixed ............................................................................................................................   3,000,000
$6,600,000

During November, Richard produced 26,000 units.  Richard used 53,500 direct labor hours in 
November at a cost of $433,350.  Actual manufacturing overhead for the month was $250,000 fixed 
and $325,000 variable.

The manufacturing overhead volume variance for November is:
A. $12,000 unfavorable
B. $10,000 unfavorable
C. $3,000 unfavorable
D. $9,000 unfavorable
E. $1,000 favorable

SUPPORTING CALCULATION:

Budget allowance based on standard hours allowed 
[(52,000 x $6) + $250,000].............................................................. $ 562,000 
Factory overhead applied at standard.......................................................            572,000 
Volume variance.......................................................................................  $      
   (10,000) favorable

C 26. The following information relates to Department 1 of Ruiz Company for the fourth quarter.  The total 
overhead variance is divided into three variances: spending, variable efficiency, and volume.

Actual total overhead (fixed plus variable)....................................... $178,500
Budget formula.................................................................................. $110,000 + $.50 per hour
Total overhead application rate......................................................... $1.50 per hour
Actual hours worked.......................................................................... 121,000

What was the spending variance in this department during the quarter?
A. $8,000 favorable
B. $4,500 favorable
C. $8,000 unfavorable
D. $4,500 unfavorable
E. none of the above
SUPPORTING CALCULATION:

Actual factory overhead.................................................... $ 178,500
Budget allowance:
Variable for actual hours 
      (121,000 x $.50)................................................... $  60,500
Fixed..........................................................................   110,000    170,500
Spending variance.............................................................  $         
   8,000 unfavorable

A 27. The following information relates to Department 1 of Ruiz Company for the fourth quarter.  The total 
overhead variance is divided into three variances: spending, variable efficiency, and volume.

Actual total overhead (fixed plus variable)....................................... $178,500
Budget formula.................................................................................. $110,000 + $.50 per hour
Total overhead application rate......................................................... $1.50 per hour
Actual hours worked.......................................................................... 121,000
Standard hours allowed for production............................................. 130,000

What was the variable efficiency variance in this department during the quarter?
A. $4,500 favorable
B. $8,000 favorable
C. $4,500 unfavorable
D. $8,000 unfavorable
E. none of the above

SUPPORTING CALCULATION:

Budget allowance for actual hours 
[(121,000 x $.50) + $110,000]................................... $ 170,500 
Budget allowance for standard hours:
Variable (130,000 x $.50).......................................... $  65,000
Fixed..........................................................................   110,000         175,000 
Variable efficiency variance..............................................  $      
   (4,500) favorable

E 28. Under the two­variance method for analyzing factory overhead, the controllable (budget) variance is 
the difference between the:
A. actual fixed factory overhead and the budgeted fixed overhead
B. budget allowance based on standard hours allowed and the factory overhead applied to 
production
C. budget allowance based on standard hours allowed and the budget allowance based on actual 
hours worked
D. actual factory overhead and the factory overhead applied to production
E. actual factory overhead and the budget allowance based on standard hours allowed

A 29. Materials usage variances are normally chargeable to the:
A. Production Department
B. Purchasing Department
C. Finished Goods Department
D. Materials Storage Department
E. Factory Storeroom Department
C 30. Todco planned to produce 3,000 units of its single product, Teragram, during November.  The 
standard specifications for one unit of Teragram include six pounds of material at $.30 per pound.  
Actual production in November was 3,100 units of Teragram.  The accountant computed a favorable 
materials purchase price variance of $380 and an unfavorable materials quantity variance of $120.  
Based on these variances, one could conclude that:
A. more materials were purchased than were used
B. more materials were used than were purchased
C. the actual cost of materials was less than the standard cost
D. the actual usage of materials was less than the standard allowed
E. actual cost and usage of materials were both less than standard

D 31. Information on Duke Co.'s direct material costs for May is as follows:

Actual quantity of direct materials purchased and used................................................ 30,000 lbs.


Actual cost of direct materials....................................................................................... $84,000
Unfavorable direct materials usage variance................................................................. 3,000
Standard quantity of direct materials allowed for May production............................... 29,000 lbs.

For the month of May, Duke's direct materials price variance was:
A. $2,800 favorable
B. $2,800 unfavorable
C. $6,000 unfavorable
D. $6,000 favorable
E. none of the above

SUPPORTING CALCULATION:

$3,000 = x (30,000 ­ 29,000)
1,000 x = $3,000
x = $3
y = $2.80 ­ $3.00(30,000)
y = ($6,000) favorable

A 32. A company uses a standard cost system to account for its only product.  The materials standard per 
unit was 4 lbs. at $5.10 per lb.  Operating data for April were as follows:

Material used.................................................................................................................. 7,800 lbs.


Cost of material used..................................................................................................... $40,950
Number of finished units produced............................................................................... 2,000

The material usage variance for April was:
A. $1,020 favorable
B. $1,050 favorable
C. $1,170 unfavorable
D. $1,200 unfavorable
E. none of the above

INTRO TO COST ACCOUNTING:
B 1. An organizational concept that groups business functions around resources, 
processes, and human interrelations is the:
A. resources function
B. functionalteamwork concept
C. processes function
D. linestaff concept 
E. matching concept

E 2. The measurement of performance and the control of costs is aided the most by:
A. organizational charts
B. continuous supervision
C. preparation for the future
D. planning 
E. budgets and standards

A 3. All of the following are abbreviations for systems or processes that represent 
changes in manufacturing technology, except:
A. CMA
B. JIT
C. CIM
D. CAD
E. FMS

D 4. The process of providing individuals with the authority to carry out their assigned 
responsibilities is referred to as:
A. control circuit
B. objective setting
C. accountability
D. delegation
E. linestaff organization

A 5. The department that uses pertinent cost data to determine products that are most 
profitable and sales policies is: 
A. Marketing
B. Manufacturing
C. Treasury
D. Legal
E. Cost

10
B 6. Examples of nonroutine planning include all of the following, except:
A. responses to the appearance of new competition
B. estimating the collection of receivables during the next month for the 
purpose of making investment/borrowing decisions
C. responses to a proposed government regulation of the industry
D. responses to a significant change in consumer tastes
E. none of the above

B 7. The coordinated development of a company's organization with the cost and 
budgetary system will lead to an approach to accounting and reporting called:
A. functionalteamwork system
B. responsibility accounting
C. linestaff organization
D. controllable segmentation
E. superiorsubordinate relationship

E 8. The organizational group that advises or performs technical functions of an 
enterprise is the:
A. line
B. function
C. team
D. executive management
E. staff

A 9. The business function in the functionalteamwork concept of management that 
deals with activities such as product design, research and development, purchasing, 
manufacturing, advertising, marketing, and billing is the:
A. processes function
B. executive function
C. resources function
D. staff
E. human interrelations function

E 10. Pronouncements of the Cost Accounting Standards Board adhere to the concept 
of:
A. indirect costing
B. common costing
C. direct costing
D. standard costing
E. full costing

B 11. The professional certification developed by the IMA indicating professional 
competence in the management accounting field is the:
A. CIA
B. CMA
12 Chapter 1
C. CA
D. CPA
E. CPM
C 12. The plans that are sufficiently detailed to permit the preparation of budgeted 
financial statements for the entity as of a future date are:
A. strategic plans
B. medium­range plans
C. short­range plans
D. long­range plans
E. none of the above

B 13. All of the following are organizations in the private sector that influence the 
development of cost accounting theory and practice except:
A. FEI
B. IRS
C. AICPA
D. FASB
E. IMA

D 14. Budgeting plays an important role in influencing individual and group behavior at
all of the following stages of the management process, except:
A. setting goals
B. motivating desired performance
C. evaluating performance
D. computing bonuses
E. suggesting when corrective action should be taken

B 15. The functionalteamwork concept of management is structured to emphasize all of 
the following except:
A. human interrelations
B. accountability
C. resources
D. processes
E. none of the above

E 16. All of the following are examples of non­value­added activities except:
A. retrieving
B. handling
C. expediting
D. reworking
E. assembling

D 17. The department that has the responsibility for the financial administration of a 
company is:
A. Tax
B. Controller's
C. Cost
D. Treasury
E. Internal Audit

D 18. The collection, presentation, and analysis of cost data should help management 
accomplish all of the following tasks except:
A. control the physical quantities of inventory
B. determine company costs and profits for an accounting period
C. choose from among two or more alternatives that will increase revenues
D. conform to FASB reporting requirements for pensions
E. establish costing methods and procedures that permit cost reductions
D 19. In an attempt to resolve an ethical conflict in a publicly­held corporation, if the 
accountant has unsuccessfully gone to the board of directors, the next step is to:
A. go to the company president
B. go back to middle management to garner support
C. report the problem to the SEC
D. resign
E. none of the above

C 20. An organizational concept recognizing that all positions or functional divisions 
can be categorized into two groups is:
A. functionalteamwork concept
B. processes function
C. linestaff concept
D. matching concept
E. resources function

A 21. In an attempt to resolve an ethical conflict when the immediate superior is 
involved, an accountant should first:
A. go to the next higher level of management
B. report the problem to the SEC
C. resign
D. go to the company president
E. none of the above

C 22. The Standards of Ethical Conduct for Management Accountants presents fifteen 
responsibilities of the management accountant that encompass all of the following categories 
except:
A. competence
B. confidentiality
C. dependability
D. integrity
E. objectivity
SUPPORTING CALCULATION:

x = $5.10 [7,800 ­ (2,000 x 4)]
x = ($1,020) favorable

D 33. During the last three months, a manufacturer incurred an unfavorable labor efficiency variance. The 
least likely cause of this variance is:
A. substantial materials were purchased at a discount at a previously unused supplier's liquidation
B. for one week, only half of the workforce, those with the highest seniority, were called in to 
work
C. a second production line with all new personnel was started 
D. the cost­of­living adjustment for the three­month period was $.10 more per hour than expected
E. none of the above

D 34. The direct labor standards for producing a unit of a product are two hours at $10 per hour.  Budgeted 
production was 1,000 units.  Actual production was 900 units, and direct labor cost was $19,000 for 
2,000 direct labor hours.  The direct labor efficiency variance was:
A. $1,000 favorable
B. $1,000 unfavorable
C. $2,000 favorable
D. $2,000 unfavorable
E. none of the above

SUPPORTING CALCULATION:

x = $10 [2,000 ­ (900 x 2)]
x = $2,000 unfavorable

C 35. Under the two­variance method for analyzing factory overhead, the factory overhead applied to 
production is used in the computation of the:

Controllable Volume
(Budget) Variance Variance
A. yes no
B. yes yes
C. no yes
D. no no

D 36. Under the three­variance method for analyzing factory overhead, which of the following is used in 
computation of the spending variance?

Actual Factory Budget Allowance
   Overhead    Based on Actual Hours
A. no yes
B. no no
C. yes no
D. yes yes
D 37. Compute the variable efficiency variance, using the following data:

Standard labor hours per good unit produced....................................................................... 2
Good units produced............................................................................................................. 1,000
Actual labor hours used........................................................................................................ 2,100
Standard variable overhead per standard labor hour............................................................. $3
Actual variable overhead...................................................................................................... $ 6,500

A. $200 favorable
B. $200 unfavorable
C. $300 favorable
D. $300 unfavorable
E. none of the above

SUPPORTING CALCULATION:

Variable budget allowance for actual hours (2,100 x $3)................................ $ 6,300
Variable budget allowance for standard hours 
($3 x 1,000 x 2)........................................................................................          6,000
.................................................................................................................  $        
   300
..............................................................................................unfavorable

The following questions are based on materials in the Appendix to the chapter.

A 38. In the alternate three­variance method, the efficiency variance is:
A. Standard factory overhead rate x (Actual units of allocation base ­ Standard units of allocation 
base allowed)
B. Actual factory overhead incurred ­ Budget allowance based on actual hours
C. Budget allowance based on actual hours ­ (Actual hours x Factory overhead rate)
D. Budgeted fixed factory overhead ­ (Actual hours x Fixed overhead rate)
E. none of the above

D 39. The four­variance method reconciles to the two­variance method by combining which of the following
to get the controllable variance?
A. fixed efficiency variance and idle capacity variance
B. spending variance and fixed efficiency variance
C. spending variance and idle capacity variance
D. spending variance and variable efficiency variance
E. none of the above

B 40. The four­variance method reconciles to the two­variance method by combining which of the following
to get the volume variance?
A. spending variance and variable efficiency variance
B. fixed efficiency variance and idle capacity variance
C. variable efficiency variance and fixed efficiency variance
D. spending variance and idle capacity variance
E. none of the above

COST CONCEPTS:

17
C 1. A cost accounting information system necessarily should accomplish all of the 
following except:
A. reflect the division of authority so that individual managers can be held 
accountable
B. provide management with information that facilitates prompt identification of 
activities needing attention
C. be more sophisticated than is required by legal, regulatory, and contractual 
requirements
D. be tailored to give the most efficient blend of sophistication and simplicity
E. focus management's attention

A 2. Cost classifications are based on the relationship of costs to all of the following 
except:
A. ledger accounts
B. accounting periods
C. products
D. volume of production
E. manufacturing departments

B 3. Direct materials and direct labor are considered to be:
A. selling expenses
B. prime costs
C. administrative expenses
D. conversion costs
E. factory overhead

E 4. Depreciation on factory buildings and equipment is classified as:
A. selling expense
B. administrative expense
C. direct labor
D. indirect materials
E. factory overhead

18
A 5. A typical marketing expense is:
A. freight out
B. indirect labor
C. audit fees
D. uncollectible accounts expense
E. direct labor

E 6. A typical indirect labor cost for a manufacturer is:
A. sales office salaries
B. freight out
C. factory insurance
D. sales commissions
E. materials handling

D 7. Usually, a cost easy to assign accurately to a specific operating department is a:
A. standard cost
B. common cost
C. fixed cost
D. variable cost
E. joint cost

C 8. In constructing a chart of accounts, all of the following guidelines should be adhered 
to except:
A. using numbers rather than letters in coding the accounts
B. dividing charts into balance sheet accounts and income statement accounts
C. using account titles that reflect a maximum level of detail about each item
D. giving maximum information with a minimum of supplementary analysis
E. providing sufficient classification to enable cost assignment to responsible 
managers

C 9. An expense that is likely to contain both fixed and variable components is:
A. security guard wages
B. supplies
C. heat, light, and power
D. small tools
E. taxes on real estate

C 10. A type of employee whose wages are not a component of indirect labor is a(n):
A. inspector
B. supervisor
C. assembler
D. maintenance worker
E. shop clerk
20 Chapter 2
B 11. Pitino Company has a beginning inventory of direct materials on March 1 of $30,000 
and an ending inventory on March 31 of $36,000.  The following additional 
manufacturing cost data were available for the month of March:

Direct materials purchased.................................................................... $84,000
Direct labor........................................................................................... 60,000
Factory overhead................................................................................... 80,000

During March, prime cost added to production was:
A. $140,000
B. $138,000
C. $144,000
D. $150,000
E. none of the above

SUPPORTING CALCULATION:  

$84,000 + $60,000 ­ ($36,000 ­ $30,000) = $138,000

C 12. Pitino Company has a beginning inventory of direct materials on March 1 of $30,000 
and an ending inventory on March 31 of $36,000.  The following additional 
manufacturing cost data were available for the month of March:

Direct materials purchased.................................................................... $84,000
Direct labor........................................................................................... 60,000
Factory overhead................................................................................... 80,000

During March, conversion cost added to production was:
A. $80,000
B. $144,000
C. $140,000
D. $138,000
E. none of the above

SUPPORTING CALCULATION:  $60,000 + $80,000 = $140,000

A 13. The term "variable costs" refers to:
A. all costs whose total amounts change in proportion to changes in activity within
a relevant range
B. all costs that are likely to respond to the amount of attention devoted to them by
a specified manager
C. all costs that are associated with marketing, shipping, warehousing, and billing 
activities
D. all costs that do not change in total for a given period and relevant range, but 
become progressively smaller on a perunit basis as volume increases
E. all manufacturing costs incurred to produce units of output
C 14. The following statement that best describes a fixed cost is:
A. it may change in total when such change depends on production within the 
relevant range
B. it increases on a perunit basis as production increases
C. it decreases on a per­unit basis as production increases
D. it may change in total when such change is related to changes in production
E. it is constant per unit of production

A 15. The term "relevant range" as used in cost accounting means the range over which:
A. cost relationships are valid
B. production may vary
C. relevant costs are incurred
D. costs may fluctuate
E. none of the above

B 16. When the number of units manufactured increases, the most significant change in 
average unit cost will be reflected as:
A. a decrease in the variable element
B. a decrease in the nonvariable element
C. an increase in the semivariable element
D. an increase in the variable element
E. an increase in the nonvariable element

C 17. Within a relevant range, the amount of variable cost per unit:
A. moves in the same direction as fixed cost per unit
B. differs at each production level
C. remains constant at each production level
D. increases as production increases
E. decreases as production increases

B 18. The term "prime costs" refers to:
A. the sum of direct labor costs and all factory overhead costs
B. the sum of direct materials costs and direct labor costs
C. manufacturing costs incurred to produce units of output
D. all costs associated with manufacturing other than direct labor and direct 
materials costs
E. cost standards that are predetermined and should be attained

B 19. The term "conversion costs" refers to:
A. costs that are associated with marketing, shipping, warehousing, and billing 
activities
B. the sum of direct labor costs and all factory overhead costs
C. the sum of direct materials costs and direct labor costs
D. manufacturing costs incurred to produce units of output
E. all costs associated with manufacturing other than direct labor costs and direct 
materials costs
A 20. Direct labor is a:

Conversion Cost Manufacturing Cost Prime Cost


A. Yes Yes Yes
B. No Yes Yes
C. No No No
D. No No Yes
E. Yes Yes No

C 21. A factory overhead cost:
A. is a direct cost
B. is a prime cost
C. can be a variable cost or a fixed cost
D. can only be a fixed cost
E. includes all factory labor

A 22. Prime cost and conversion cost share what common element of total cost?
A. direct labor
B. commercial expense
C. variable overhead
D. fixed overhead
E. direct materials

E 23. Factory overhead includes:
A. indirect materials but not indirect labor
B. indirect labor but not indirect materials
C. prime costs
D. all manufacturing costs
E. all manufacturing costs, except direct materials and direct labor

C 24. Indirect materials are a(n):
A. fixed cost
B. irrelevant cost
C. factory overhead cost
D. direct cost
E. prime cost

C 25. Wages of the security guard for a small plant are an example of:

Fixed Factory
Indirect Labor    Overhead  
A. No Yes
B. No No
C. Yes Yes
D. Yes No
E. none of the above
B 26. Wages paid to factory machine operators of a manufacturing plant are an element of:

Prime Cost Conversion Cost
A. Yes No
B. Yes Yes
C. No No
D. No Yes
E. none of the above

E 27. Common costs are:
A. costs that occur when the production of one product is possible only if one or 
more other products are manufactured at the same time
B. intended to benefit future periods
C. variable in direct proportion to the level of production
D. chargeable directly to the product
E. costs of facilities or services employed by two or more operations

D 28. Joint costs are:
A. direct costs
B. costs of facilities or services employed by two or more operations
C. revenue expenditures
D. incurred when the production of one product is possible only if other products 
are produced at the same time
E. always variable

B 29. All of the following are examples of nonfinancial performance measures except:
A. the number of defective units produced
B. the gross margin on a product line income statement
C. hours of machine downtime
D. number of days on schedule
E. weight of scrap material produced

E 30. Reasons for the increased attention being given to nonfinancial performance 
measures include:
A. dissatisfaction with exclusive reliance on financial measures
B. dissatisfaction with financial measures of plant utilization
C. dissatisfaction with financial measures of processing efficiency
D. dissatisfaction with the slow pace at which a company's data processing system 
can modify traditional financial measures
E. all of the above

E 31. Of the following items, a cost object is:
A. a unit of product
B. a customer order
C. a project
D. a division of the company
E. all of the above
E 32. General corporate­level costs, such as bond interest and taxes, would be readily 
traceable to:
A. each unit of product
B. each division of the company
C. each batch of production
D. all units of product ever produced
E. none of the above

D 33. A revenue expenditure is one that: 
A. varies with the volume of production
B. is intended to benefit future periods
C. is reported as an asset
D. benefits the current period only
E. remains the same in total as production changes

B 34. An example of a cost that is irrelevant to a future decision is a(n):
A. differential cost
B. sunk cost
C. out­of­pocket cost
D. opportunity cost
E. variable cost

A 35. The measures in a balanced scorecard’s growth and learning perspective attempt to 
report on:
A. three kinds of intangible resources: human capital, information, and the 
alignment of incentives
B. the organization’s most important work—work in which the organization must 
excel in order to successful
C. the final results that are most important to the owners of the organization, and 
the rates of improvement in those results
D. the extent to which the organization is creating and sustaining desirable 
customer relationships
E. all of the above

E 36. An example of a cost that is irrelevant to a future decision is a(n):
A. communicating plans
B. focusing attention
C. implementing strategy
D. monitoring progress toward strategic objectives
E. all of the above

A 37. When a balanced scorecard represents a series of predictions telling how 
management intends for the organization to succeed, the predictions are in a sequence
that begins with:
A. growth and learning
B. innovation
C. customer service
D. customer satisfaction
E. investment opportunity

A 38. Balanced scorecards are called “balanced” because they report:
A. both leading and lagging measures
B. both balance sheets and income statements
C. both historical costs and replacements costs
D. both original costs and book values
E. all of the above

COST BEHAVIORS:

D 1. Expenses that require a series of payments over a long period of timeCsuch as long­term debt and 
lease rentalsCare frequently known as:
A. programmed fixed expenses
B. avoidable expenses
C. variable expenses
D. committed fixed expenses
E. normal capacity expenses

C 2. A mathematical technique used to fit a straight line to a set of plotted points is:
A. integral calculus
B. the EOQ model
C. the method of least squares
D. linear programming
E. PERT network analysis

E 3. One advantage of using multiple regression analysis is that:
A. computations are simplified
B. only two data points need be considered
C. a twodimensional graph may be used to show cost relationships
D. costs may be grouped into one independent variable
E. the effects of several variables on costs may be analyzed
 
B 4. The coefficient of determination indicates:
A. causal relationships among costs and other factors
B. the percentage of explained variance in the dependent variable
C. the linear relationship between two variables
D. whether several variables fluctuate
E. the size of the standard deviation

30
E 5. Hoyden Co. developed the following equation to predict certain components of its budget for the 
coming period:

Costs = $50,000 + ($5 x direct labor hours)

The $5 would approximate:
A. total cost
B. direct labor rate per hour
C. fixed cost per direct labor hour
D. the coefficient of determination
E. variable costs per direct labor hour

E 6. When cost relationships are linear, total variable manufacturing costs will vary in proportion to 
changes in:
A. machine hours
B. direct labor hours
C. total material cost
D. total overhead cost
E. volume of production
 
B 7. The term "relevant range" as used in cost accounting means the range over which:
A. relevant costs are incurred
B. cost relationships are valid
C. costs may fluctuate
D. sales volume fluctuates
E. production may vary
 
E 8. Within a relevant range, the amount of fixed cost per unit:
A. differs at each production level on a perunit basis
B. remains constant in total
C. decreases as production increases on a perunit basis
D. increases as production decreases on a perunit basis
E. all of the above

C 9. The following relationships pertain to a year's budgeted activity for Buckeye Company:

High   Low   
Direct labor hours................................................................ 400,000 300,000
Total costs............................................................................ $154,000 $129,000

What are the budgeted fixed costs for the year?
A. $100,000
B. $25,000
C. $54,000
D. $75,000
E. none of the above
SUPPORTING CALCULATION:

High..................................................................................... $ 154,000 400,000


Low......................................................................................         129,000 300,000
Difference............................................................................  $          25,000 100,000

Variable rate = $25,000 )  100,000 = $.25/direct labor hour
Fixed cost = $154,000 ­ $.25(400,000) = $54,000

B 10. Maintenance expenses of a company are to be analyzed for purposes of constructing a flexible budget. 
Examination of past records disclosed the following costs and volume measures:

High Low
Cost per month..................................................................... $39,200 $32,000
Machine hours...................................................................... 24,000 15,000

Using the highlow method of analysis, the estimated variable cost per machine hour is:
A. $12.50
B. $0.80
C. $0.08
D. $1.25
E. none of the above

SUPPORTING CALCULATION:

High..................................................................................... $ 39,200 24,000


Low......................................................................................         32,000 15,000
Difference............................................................................  $    
      7,200   9,000

Variable rate = $7,200 )  9,000 = $.80/machine hour

D 11. A company allocates its variable factory overhead based on direct labor hours. During the past three 
months, the actual direct labor hours and the total factory overhead allocated were as follows:

October November December


Direct labor hours...................................... 2,500 3,000 5,000
Total factory 
overhead allocated.............................. $80,000 $75,000 $100,000

Based upon this information, the estimated variable cost per direct labor hour was:
A. $.125
B. $12.50
C. $.08
D. $8
E. none of the above
SUPPORTING CALCULATION:

High..................................................................................... $ 100,000 5,000


Low......................................................................................          80,000 2,500
Difference............................................................................  $          20,000 2,500

Variable rate = $20,000 )  2,500 = $8.00/direct labor hour

A 12. The technique that can be used to determine the variable and fixed portions of a company's costs is:
A. scattergraph method
B. poisson analysis
C. linear programming
D. game theory
E. queuing theory

A 13. The number of variables used in simple regression analysis is:
A. two
B. three
C. more than three
D. three or less
E. one

C 14. Multiple regression analysis:
A. is not a sampling technique
B. involves the use of independent variables only
C. assumes that the independent variables are not correlated
D. establishes a causeandeffect relationship
E. all of the above

E 15. For a simple regression­analysis model that is used to allocate factory overhead, an internal auditor 
finds that the intersection of the line of best fit for the overhead allocation on the y­axis is $50,000. 
The slope of the trend line is .20. The independent variable, factory wages, amounts to $900,000 for 
the month. What is the estimated amount of factory overhead to be allocated for the month?
A. $910,000
B. $950,000
C. $ 50,000
D. $180,000
E. $230,000

SUPPORTING CALCULATION:

Factory overhead = $50,000 + .2($900,000) = $230,000
A 16. As a result of analyzing the relationship of total factory overhead to changes in machine hours, the 
following relationship was found:

    y bar = $1,000 + $2 x bar

This equation was probably found by using the mathematical techniques called:
A. simple regression analysis
B. dynamic programming
C. linear programming
D. multiple regression analysis
E. none of the above

A 17. As a result of analyzing the relationship of total factory overhead to changes in machine hours, the 
following relationship was found:

y bar = $1,000 + $2 x bar

The y bar in the equation is an estimate of:
A. total factory overhead
B. total fixed costs
C. total machine costs
D. total variable costs
E. none of the above

C 18. As a result of analyzing the relationship of total factory overhead to changes in machine hours, the 
following relationship was found:

y bar = $1,000 + $2 x bar

The $2 in the equation is an estimate of:
A. fixed costs per machine hour
B. total fixed costs
C. variable costs per machine hour
D. total variable costs
E. none of the above

D 19. As a result of analyzing the relationship of total factory overhead to changes in machine hours, the 
following relationship was found:

y bar = $1,000 + $2 x bar

The use of such a relationship of total factory overhead to changes in machine hours is said to be valid 
only within the relevant range, which means:
A. within the range of reasonableness as judged by the department supervisor
B. within the budget allowance for overhead
C. within a reasonable dollar amount for machine costs
D. within the range of observations of the analysis
E. none of the above
C 20. A measure of the extent to which two variables are related linearly is referred to as:
A. sensitivity analysis
B. inputoutput analysis
C. coefficient of correlation
D. causeeffect ratio
E. costbenefit analysis

C 21. The appropriate range for the coefficient of correlation (r) is:
A. infinity # r # infinity
B. 0 # r # 1
C. 1 # r # 1
D. 100 # r # 100
E. none of the above

A 22. The covariation between two variables, such as direct labor hours and electricity expense, can best be 
measured by:
A. correlation analysis
B. simple regression analysis
C. multiple regression analysis
D. high­low method
E. scattergraph method

B 23. The quantitative method that will separate a semivariable cost into its fixed and variable components 
with the highest degree of precision is:
A. simplex method
B. least squares method
C. scattergraph method
D. account analysis
E. highlow method

A 24. If the coefficient of correlation between two variables is zero, a scatter diagram of these variables 
would appear as:
A. random points
B. a least squares line that slopes up to the right
C. a least squares line that slopes down to the right
D. under this condition, a scatter diagram could not be plotted on a graph
E. none of the above

D 25. Multiple regression analysis involves the use of:

Dependent Independent
 Variables    Variables  
A. 1 none
B. 1> 1
C. 1> 1>
D. 1 1>
C 26. A company using regression analysis to correlate income to a variety of sales indicators found that the 
relationship between the number of sales managers in a territory and net income for the territory had a 
correlation coefficient of 1. The best description of this situation is:
A. that more sales managers should be hired
B. imperfect negative correlation
C. perfect inverse correlation
D. no correlation
E. perfect positive correlation

B 27. The correlation coefficient that indicates the weakest linear association between two variables is:
A. 0.73
B. 0.11
C.  0.12
D.  0.35
E.  0.72

B 28. If regression was applied to the data shown in Figure 3­1, the coefficients of correlation and 

determination would indicate the existence of a:

A. low linear relationship, high explained variation ratio
B. high inverse linear relationship, high explained variation ratio
C. high direct linear relationship, high explained variation ratio
D. high inverse linear relationship, low explained variation ratio
E. none of the above

A 29. Omitting important variables from the multiple regression is referred to as a(n):
A. specification error
B. autocorrelation
C. confidence loss
D. homoscedastic error
E. heteroscedastic error

E 30. When two or more independent variables are correlated with one another, the condition is referred to 
as:
A. serial correlation
B. autocorrelation
C. heteroscedacity
D. homoscedacity
E. multicollinearity
A 31. A large value for standard error of the estimate indicates that:
A. the actual cost will likely vary greatly from the estimated cost as portrayed by the regression 
line
B. the actual cost will be greater than the estimate cost as portrayed by the regression line
C. the actual cost will be less than the estimate cost as portrayed by the regression line
D. the actual cost will likely vary little from the estimated cost as portrayed by the regression line
E. none of the above

D 32. The confidence interval represents:
A. the percentage of variance in the dependent variable as explained by the independent variable
B. the measure of the extent to which variables are related linearly
C. the standard deviation about the regression line
D. a range of values within which the dependent variable is expected to fall a certain percentage of
the time
E. none of the above

C 33. When the distribution of observations around the regression line is uniform for all values of the 
independent variable, it is:
A. heteroscedastic
B. serially correlated
C. homoscedastic
D. autocorrelated
E. none of the above

E 34. Expenses that are fixed at management's discretion at a certain level for the period are referred to as:
A. committed fixed costs
B. mixed costs
C. opportunity costs
D. sunk costs
E. programmed fixed costs

A 35. The separation of fixed and variable costs is necessary for all of the following purposes except:
A. absorption costing and net income analysis
B. direct costing and contribution margin analysis
C. break­even and cost­volume­profit analysis
D. differential and comparative cost analysis
E. capital budgeting analysis

COST ACCUMULATION:

E 1. The tiein between general accounts and cost accounts is often discussed with accounting procedures. 
An example of a general account is:
A. Materials 
B. Work in Process
C. Factory Overhead Control
D. Finished Goods
E. Accumulated Depreciation

C 2. One feature of a standard cost system is that:
A. selection of the cost unit becomes simplified
38
B. predetermined amounts are ignored
C. an analysis of cost variances is facilitated
D. historical costs are recorded as they are incurred
E. reports are delayed until operations have been performed

A 3. An industry that would most likely use job order costing procedures is: 
A. road building
B. fertilizer manufacturing
C. flour milling
D. petroleum refining
E. textile manufacturing

D 4. An industry that would most likely use process costing procedures is: 
A. musical instrument manufacturing
B. construction
C. aircraft
D. chemicals
E. office equipment

A 5. Supplies needed for use in the factory are issued on the basis of: 
A. materials requisitions
B. time tickets
C. factory overhead analysis sheets
D. clock cards
E. purchase invoices
40 Chapter 4

D 6. Finished Goods is debited and Work in Process is credited for a: 
A. transfer of materials to the factory
B. return of unused materials from the factory
C. purchase of goods on account
D. transfer of completed production
E. transfer of completed goods out of the factory

B 7. The best cost accumulation procedure to use when many batches, each differing as to product 
specifications, are produced is: 
A. absorption
B. job order
C. process
D. actual
E. standard

A 8. Job order costs are most useful for:
A. determining the cost of a specific project
B. determining the labor cost involved in production
C. determining inventory valuation using lifo
D. estimating overhead costs
E. controlling indirect costs of future production

E 9. Under a job order cost system, the dollar amount of the entry to transfer the inventory from Finished 
Goods to Cost of Goods Sold is the sum of the costs charged to all jobs: 
A. completed during the period
B. started in process during the period
C. in process during the period
D. completed and sold during the period
E. sold during the period

A 10. The industry most likely to use job order costing in accounting for costs is:
A. accounting firms
B. textile manufacturer
C. paint manufacturer
D. oil refinery
E. none of the above

A 11. Job order cost accounting systems and process accounting systems differ in the way: 
A. costs are traced to cost objects
B. orders are taken and in the number of units in the orders
C. product profitability is determined and compared with planned costs 
D. manufacturing processes can be accomplished and in the number of production runs that may 
be performed in a year
E. none of the above

D 12. In a job order cost system, the distribution of direct labor costs usually are recorded as an increase in: 
A. Cost of Goods Sold
B. Factory Overhead Control
C. Finished Goods
D. Work in Process
E. none of the above
C 13. Process costing techniques should be used in assigning costs to products:
A. if the product is manufactured on the basis of each order received
B. when production is only partially completed during the accounting period
C. if the product is composed of massproduced homogeneous units
D. whenever standard costing techniques should not be used
E. none of the above

A 14. A characteristic of a process costing system is:
A. partially processed inventory is restated in terms of completed units
B. costs are accumulated by order
C. it is used by a company manufacturing custom machinery
D. standard costs are not applicable
E. none of the above

C 15. The industry most likely to use process costing in accounting for costs is:
A. road builder
B. electrical contractor
C. airlines
D. automobile repair shop
E. none of the above

B 16. In the computation of manufacturing cost per equivalent unit, the weighted average method of process 
costing considers:
A. current costs only
B. current costs plus cost of beginning work in process inventory
C. current costs plus cost of ending work in process inventory
D. current costs less cost of beginning work in process inventory
E. none of the above

B 17. The element of manufacturing cost that supports time tickets is:
A. materials
B. labor
C. factory overhead
D. all of the above
E. none of the above

C 18. The element of manufacturing cost that supports depreciation schedules is:
A. materials
B. labor
C. factory overhead
D. all of the above
E. none of the above

D 19. Work in Process is debited and Materials is credited for:
A. indirect materials requisitioned to production
B. the completion of work in process
C. the sale of completed product
D. direct materials requisitioned to production
E. materials returned to the storeroom
E 20. Factory Overhead Control is debited and Work in Process is credited for:
A. indirect materials requisitioned to production
B. the completion of work in process
C. the sale of completed product
D. direct materials requisitioned to production
E. none of the above

A 21. Products, operations, and processes costed on the basis of predetermined quantities of resources to be 
used and predetermined prices of those resources are distinguishing characteristics of which:
A. standard cost system
B. historical cost system
C. process cost system
D. job order cost system
E. backflush cost system

D 22. The tax requirement that certain purchasing and storage costs be allocated to inventory is known as:
A. backflush costing
B. postdeduction
C. justintime
D. super absorption
E. none of the above

C 23. The manufacturing systems characterized by short setup times, moderate to low lead times, and very 
low direct labor cost is:
A. manual systems
B. fixed automation systems
C. flexible manufacturing systems
D. process cost systems
E. job order cost systems

E 24. The cost accounting system noted for its lack of detailed tracking of work in process during the 
accounting period is:
A. process costing
B. job order costing
C. standard costing
D. actual costing
E. backflush costing

D 25. Ziffel Company had the following account balances and results from operations for the month of July: 
direct materials consumed, $10,400; direct labor, $8,000; factory overhead, $8,800; July 1, work in 
process inventory, $2,400; July 31, work in process inventory, $1,800; finished goods inventory, July 
1, $1,200; finished goods inventory, July 31, $1,000.  The total manufacturing cost for the month of 
July was:
A. $27,800
B. $28,000
C. $18,400
D. $27,200
E. none of the above

SUPPORTING CALCULATION:  $10,400 + $8,000 + $8,800 = $27,200
C 26. Ziffel Company had the following account balances and results from operations for the month of July: 
direct materials consumed, $10,400; direct labor, $8,000; factory overhead, $8,800; July 1, work in 
process inventory, $2,400; July 31, work in process inventory, $1,800; finished goods inventory, July 
1, $1,200; finished goods inventory, July 31, $1,000.  The cost of goods manufactured was:
A. $27,200
B. $28,000
C. $27,800
D. $26,600
E. none of the above

SUPPORTING CALCULATION:  $27,200 + $2,400 ­ $1,800 = $27,800

B 27. Ziffel Company had the following account balances and results from operations for the month of July: 
direct materials consumed, $10,400; direct labor, $8,000; factory overhead, $8,800; July 1, work in 
process inventory, $2,400; July 31, work in process inventory, $1,800; finished goods inventory, July 
1, $1,200; finished goods inventory, July 31, $1,000.  The cost of goods sold was:
A. $27,200
B. $28,000
C. $27,800
D. $27,600
E. none of the above

SUPPORTING CALCULATION:  $27,800 + $1,200 ­ $1,000 = $28,000

A 28. A cost system where only the variable manufacturing costs are allocated to production is:
A. direct costing
B. prime costing
C. absorption costing
D. standard costing
E. none of the above

D 29. A cost system where all manufacturing cost elements are allocated to production is:
A. direct costing
B. prime costing
C. standard costing
D. full absorption costing
E. none of the above

C 30. A cost system where only direct material and direct labor costs are allocated to production is:
A. direct costing
B. standard costing
C. prime costing
D. full absorption costing
E. none of the above
B 31. The manufacturing systems that are characterized by very high setup times, moderate lead times, and 
high direct labor cost are:
A. flexible manufacturing systems
B. fixed automation systems
C. manual systems
D. backflush systems
E. none of the above
JOB ORDER:

A 1. Under job order cost accumulation, the factory overhead control account controls:
A. factory overhead analysis sheets
B. all general ledger subsidiary accounts
C. job order cost sheets 
D. cost reports by processes
E. materials inventories 
B 2. Supplies needed for use in the factory are issued on the basis of:  
A. job cost sheets
B. materials requisitions
C. time tickets
D. factory overhead analysis sheets
E. clock cards

B 3. Finished Goods is debited and Work in Process is credited for a:
A. transfer of completed goods out of the factory
B. transfer of completed production to the finished goods storeroom
C. purchase of goods on account
D. transfer of materials to the factory
E. return of unused materials from the factory

A 4. In job order costing, when materials are returned to the storekeeper that were previously issued to the 
factory for cleaning supplies, the journal entry should be made to:
A. Materials
Factory Overhead
B. Materials
Work in Process
C. Purchases Returns
Work in Process
D. Work in Process
Materials
E. Factory Overhead
Work in Process

47
48 Chapter 5

A 5. Under a job order cost system, the dollar amount of the entry to transfer the inventory from Work in 
Process to Finished Goods is the sum of the costs charged to all jobs:
A. completed during the period
B. started in process during the period
C. in process during the period
D. completed and sold during the period
E. none of the above

B 6. When a manufacturing company has a highly automated plant producing many different products, 
probably the most appropriate basis of applying factory overhead costs to Work in Process is:
A. units processed
B. machine hours
C. direct labor hours
D. direct labor dollars
E. none of the above

A 7. Cherokee Company applies factory overhead on the basis of direct labor hours.  Budget and actual 
data for direct labor and overhead for the year are as follows:

Budget  Actual 
Direct labor hours............................................................................. 600,000 650,000
Factory overhead costs..................................................................... $720,000 $760,000

The factory overhead for Cherokee for the year is:
A. overapplied by $20,000
B. overapplied by $40,000
C. underapplied by $20,000
D. underapplied by $40,000
E. neither underapplied nor overapplied

SUPPORTING CALCULATION:
C 8. At the end of the year, Paola Company had the following account balances after applied factory 
overhead had been closed to Factory Overhead Control:

Factory Overhead Control......................................................................................... $ 1,000 CR
Cost of Goods Sold.................................................................................................... 980,000 DR
Work in Process......................................................................................................... 38,000 DR
Finished Goods.......................................................................................................... 82,000 DR

The most common treatment of the balance in Factory Overhead Control would be to:
A. carry it as a deferred credit on the balance sheet
B. report it as miscellaneous operating revenue on the income statement
C. credit it to Cost of Goods Sold
D. prorate it between Work in Process and Finished Goods
E. prorate it among Work in Process, Finished Goods, and Cost of Goods Sold

B 9. Overapplied factory overhead would result if:
A. the plant were operated at less than normal capacity
B. factory overhead costs incurred were less than costs charged to production
C. factory overhead costs incurred were unreasonably large in relation to units produced
D. factory overhead costs incurred were greater than costs charged to production
E. a firm incurred a significant amount of overhead

A 10. The Waitkins Company estimated Department A's overhead at $255,000 for the period based on an 
estimated volume of 100,000 direct labor hours.  At the end of the period, the factory overhead control 
account for Department A had a balance of $265,500; actual direct labor hours were 105,000.  What 
was the over­ or under­applied overhead for the period?
A. $2,250 
B. $(2,250)
C. $15,000 
D. $(15,000)
E. $(5,000)

SUPPORTING CALCULATION:
D 11. Howell Corporation has a job order cost system.  The following debits (credits) appeared in Work in 
Process for the month of July:

July 1, balance................................................................................................................ $   12,000 
July 31, direct materials.................................................................................................. 40,000 
July 31, direct labor........................................................................................................ 30,000 
July 31, factory overhead................................................................................................ 27,000 
July 31, to finished goods............................................................................................... (100,000)

Howell applies overhead to production at a predetermined rate of 90% based on the direct labor cost.  
Job 1040, the only job still in process at the end of July, has been charged with factory overhead of 
$2,250.  What was the amount of direct materials charged to Job 1040?
A. $6,750
B. $2,250
C. $2,500
D. $4,250
E. $9,000

SUPPORTING CALCULATION:

Job 1040 = $12,000 + $40,000 + $30,000 + $27,000 ­ $100,000 = $9,000

E 12. Valentino Corporation makes aluminum fasteners.  Among Valentino's 19­ manufacturing costs were:

Wages and salaries:
Machine operators.................................................................................................. $80,000
Factory supervisors................................................................................................  30,000
Machine mechanics................................................................................................  20,000

Direct labor amounted to:
A. $50,000
B. $100,000
C. $110,000
D. $130,000
E. none of the above
B 13. Rudolpho Corporation makes aluminum fasteners.  Among Rudolpho's 19­­ manufacturing costs were:

Materials and supplies:
Aluminum..................................................................................................................... $400,000
Machine parts............................................................................................................... 18,000
Lubricants for machines............................................................................................... 5,000

Direct materials amounted to:
A. $23,000
B. $400,000
C. $405,000
D. $418,000
E. $423,000

C 14. Selected cost data (in thousands) concerning the past fiscal year's operations of the Moscow 
Manufacturing Company are presented below.

         Inventories       
Beginning Ending
Materials........................................................................................... $75  $  85
Work in process................................................................................ 80 30
Finished goods.................................................................................. 90 110

Materials used, $326
Total manufacturing costs charged to production during the year (including direct materials, direct 
labor, and factory overhead applied at the rate of 60% of direct labor cost), $686
Cost of goods available for sale, $826
Selling and general expenses, $25

The cost of direct materials purchased during the year amounted to:
A. $360
B. $316
C. $336
D. $411
E. none of the above

SUPPORTING CALCULATION:  $326 + $85 ­ $75 = $336
C 15. Selected cost data (in thousands) concerning the past fiscal year's operations of the Moscow 
Manufacturing Company are presented below.

         Inventories        
Beginning Ending
Materials........................................................................................... $75 $  85
Work in process................................................................................ 80 30
Finished goods.................................................................................. 90 110

Materials used, $326
Total manufacturing costs charged to production during the year (including direct materials, direct 
labor, and factory overhead applied at the rate of 60% of direct labor cost), $686
Cost of goods available for sale, $826
Selling and general expenses, $25

Direct labor costs charged to production during the year amounted to:
A. $216
B. $135
C. $225
D. $360
E. none of the above

SUPPORTING CALCULATION: $686 = $326 + x + .6x
x = $225

A 16. Selected cost data (in thousands) concerning the past fiscal year's operations of the Moscow 
Manufacturing Company are presented below.

          Inventories        
Beginning Ending
Materials........................................................................................... $75 $   85
Work in process................................................................................ 80 30
Finished goods.................................................................................. 90 110

Materials used, $326
Total manufacturing costs charged to production during the year (including direct materials, direct 
labor, and factory overhead applied at the rate of 60% of direct labor cost), $686
Cost of goods available for sale, $826
Selling and general expenses, $25

The cost of goods manufactured during the year was:
A. $736
B. $716
C. $636
D. $766
E. none of the above

SUPPORTING CALCULATION:  $80 + $686 ­ $30 = $736
A 17. Selected cost data (in thousands) concerning the past fiscal year's operations of the Moscow 
Manufacturing Company are presented below.

         
Inventories        
Beginning Ending
Materials........................................................................................... $75 $  85
Work in process................................................................................ 80 30
Finished goods.................................................................................. 90 110

Materials used, $326
Total manufacturing costs charged to production during the year (including direct materials, direct 
labor, and factory overhead applied at the rate of 60% of direct labor cost), $686
Cost of goods available for sale, $826
Selling and general expenses, $25

The cost of goods sold during the year was:
A. $716
B. $691
C. $801
D. $736
E. none of the above

SUPPORTING CALCULATION:  $90 + $736 ­ $110 = $716

A 18. J. D. Doonesbury Company manufactures tools to customer specifications.   The following data 
pertain to Job 1501 for April:

Direct materials used....................................................................................................... $ 4,200
Direct labor hours worked............................................................................................... 300
Direct labor rate per hour................................................................................................. $ 8.00
Machine hours used......................................................................................................... 200
Applied factory overhead rate per machine hour............................................................ $ 15.00

What is the total manufacturing cost recorded on Job 1501 for April?
A. $9,600
B. $10,300
C. $11,100
D. $5,400
E. $8,800

SUPPORTING CALCULATION:  $4,200 + (300 x $8) + (200 x $15) = $9,600

C 19. In service businesses using job order costing, the most commonly used base for applying overhead to 
jobs is:
A. machine hours
B. direct materials consumed
C. direct labor cost
D. meals, travel, and entertainment
E. none of the above
A 20. In service businesses using job order costing, the hourly rate used to charge costs to a job usually 
includes:
A. both labor and overhead cost
B. labor cost only
C. overhead cost only
D. labor, overhead, and miscellaneous costs
E. none of the above

A 21. Work in Process is debited and Materials is credited for:
A. the issuance of direct materials into production
B. the issuance of indirect materials into production
C. the return of materials to the storeroom
D. the application of materials overhead
E. none of the above

B 22. Factory Overhead Control is debited and Payroll is credited for:
A. the recording of payroll
B. the distribution of indirect labor costs
C. the distribution of direct labor costs
D. the distribution of withholding taxes
E. none of the above

A 23. Applied Factory Overhead is debited and Factory Overhead is credited to:
A. close the estimated overhead account to actual overhead
B. record the actual factory overhead for the period
C. charge estimated overhead to all jobs worked on during the period
D. to record overapplied overhead for the period
E. none of the above

C 24. The best overhead allocation base to use in a labor­intensive manufacturing environment probably 
would be:
A. materials cost
B. machine hours
C. direct labor hours
D. units of production
E. none of the above

D 25. Finished Goods is debited and Cost of Goods Sold is credited for:
A. transfer of completed goods to the customer
B. sale of a customer order
C. return of materials to the supplier
D. return of goods by the customer
E. none of the above

PROCESS COSTING:

B 1. An equivalent unit of material or conversion cost is equal to: 
A. the prime cost
B. the amount of material or conversion cost necessary to complete one unit of production
54
C. a unit of work in process inventory
D. the amount of material or conversion cost necessary to start a unit of production into work in 
process
E. 50% of the material or conversion cost of a unit of finished goods inventory, assuming a linear 
production pattern

B 2. The product flow format where certain portions of the work are done simultaneously and then brought 
together for completion is called: 
A. applied
B. parallel
C. standard
D. selective
E. sequential

C 3. An item that does not appear on a cost of production report is: 
A. work in processCbeginning inventory
B. cumulative costs through the end of departmental production
C. finished goodsCending inventory
D. materials used in the department
E. unit costs added by the department

C 4. Goode Manufacturing has three producing departments in its factory. The ending inventory in the 
Milling Department consisted of 3,000 units. These units were 60% complete with respect to labor and
factory overhead. Materials are applied at the end of the milling process. Unit costs for the complete 
process in the Milling Department are:  materials, $1; labor, $2; and factory overhead, $3. The 
appropriate unit cost for each unit in the ending inventory is: 
A. $2
B. $5
C. $3
D. $6
E. $4

SUPPORTING CALCULATION:  60% ($2 + $3) = $3

55
56 Chapter 6

D 5. When added materials in subsequent departments result in an increase of the units produced, the unit 
transferred­in costs will: 
A. be reclassified as new materials
B. be increased to provide for the additional units
C. be accounted for under the fifo costing method
D. be decreased as they are spread over more units
E. remain unchanged

E 6. Gyro Products transferred 10,000 units to one department. An additional 3,000 units of materials were 
added in the department. At the end of the month, 7,000 units were transferred to the next department. 
There was no beginning inventory. The costs for units transferred in would be effectively allocated 
over:
A. 17,000 units
B. 3,000 units
C. 10,000 units
D. 7,000 units
E. 13,000 units

SUPPORTING CALCULATION: 7,000 units transferred out + 6,000 units in ending inventory 
= 13,000 units

E 7. A characteristic of a process costing system is that:
A. costs are accumulated by order
B. it is used by a company manufacturing custom machinery
C. standard costs are not applicable
D. it requires a lot more detailed accounting than does a job order system
E. work in process inventory is restated in terms of completed units

D 8. Transferred­in costs as used in a process cost accounting system are:
A. supervisory salaries that are transferred from an overhead cost center to a production cost center
B. ending work in process inventory of a previous process that will be used in a succeeding 
process
C. labor that is transferred from another department within the same plant instead of hiring 
temporary workers from the outside 
D. costs of the product of a previous internal process that is subsequently used in a succeeding 
internal process
E. none of the above

E 9. In a process costing system, how is the unit cost affected in a production cost report when materials 
are added in a department subsequent to the first department and the added materials result in 
additional units?
A. The first department's unit cost is increased, but it does not necessitate an adjustment of the 
transferred­in unit cost.
B. The first department's unit cost is decreased, but it does not necessitate an adjustment of the 
transferred­in unit cost.
C. The first department's unit cost is not affected.
D. The first department's unit cost is increased, which necessitates an adjustment of the 
transferred­in unit cost.
E. The first department's unit cost is decreased, which necessitates an adjustment of the 
transferred­in unit cost.
E 10. Assuming that there was no beginning work in process inventory and the ending work in process 
inventory is 50% complete as to conversion costs, the number of equivalent units as to conversion 
costs would be:
A. less than the units completed
B. more than the units completed
C. the same as the units placed in process
D. the same as the units completed
E. less than the units placed in process

A 11. An error was made in the computation of the percentage of completion of the current year's ending 
work in process inventory. The error resulted in assigning a lower percentage of completion to each 
component of the inventory than actually was the case. What is the effect of this error upon:
(1) the computation of equivalent units in total
(2) the computation of costs per equivalent unit
(3) costs assigned to cost of goods completed for the period

(1) (2) (3)


A. understate overstate overstate
B. understate understate overstate
C. overstate understate understate
D. overstate overstate understate
E. none of the above

C 12. Read, Inc. instituted a new process in October. During October, 10,000 units were started in 
Department A. Of the units started, 7,000 were transferred to Department B, and 3,000 remained in 
work in process at October 31. The work in process at October 31 was 100% complete as to material 
costs and 50% complete as to conversion costs. Materials costs of $27,000 and conversion costs of 
$39,950 were charged to Department A in October. What were the total costs transferred to 
Department B?
A. $46,900
B. $53,600
C. $51,800
D. $57,120
E. none of the above

SUPPORTING CALCULATION:

Materials unit cost = $27,000 )  (7,000 + 3,000) = $2.70
Conversion unit cost = $39,950 )  [7,000 + 50%(3,000)] = $4.70
Costs transferred = 7,000($2.70 + $4.70) = $51,800

D 13. In accounting for beginning inventory costs, the method that allows the addition of beginning 
inventory costs with costs incurred during the period is referred to as:
A. first­in, first­out
B. addition
C. last­in, first­out
D. average
E. first­in, last­out
E 14. Chicago Processing Co. uses the average costing method and reported a beginning inventory of 5,000 
units that were 20% complete with respect to materials in one department. During the month, 11,000 
units were started; 8,000 units were finished; ending inventory amounted to 8,000 units that were 60% 
complete with respect to materials. Total materials cost during the period for work in process should 
be spread over:
A. 7,200 units
B. 16,000 units
C. 11,200 units
D. 13,200 units
E. 12,800 units

SUPPORTING CALCULATION:  8,000 + .60(8,000) = 12,800 units

E 15. In determining the cost of goods transferred in from a previous department under the average cost 
method:
A. a simple average of unit costs is used
B. beginning inventory costs are separated from costs transferred in during the period
C. a first­in, first­out approach is used
D. equivalent production in ending inventory is separated from other transferred­in costs
E. a weighted average of unit costs is used

E 16. The average and fifo process costing methods differ in that the average method:
A. can be used under any cost flow assumption
B. is much more difficult to apply than the fifo method
C. requires that ending work in process inventory be stated in terms of equivalent units of 
production
D. considers the ending work in process inventory only partially complete
E. does not consider the degree of completion of beginning work in process inventory when 
computing equivalent units of production

A 17. The first step in applying the average cost method is to: 
A. add the beginning work in process costs to the current period's production costs
B. divide the current period's production costs by the equivalent units
C. subtract the beginning work in process costs from the current period's production costs
D. A and B
E. B and C

C 18. Beginning work in process was 60% complete as to conversion costs, and ending work in process was 
45% complete as to conversion costs. The dollar amount of the conversion cost included in ending 
work in process (using the average cost method) is determined by multiplying the average unit 
conversion costs by what percentage of the total units in ending work in process?
A. 60%
B. 55%
C. 45%
D. 522%
E. 100%
C 19. Dover Corporation's production cycle starts in the Mixing Department. The following information is 
available for April:

Units 
Work in process, April 1 (50% complete)...................................................................... 40,000
Started in April............................................................................................................... 240,000
Work in process, April 30 (60% complete).................................................................... 25,000

Materials are added at the beginning of the process in the Mixing Department. Using the average cost 
method, what are the equivalent units of production for the month of April?

Materials    Conversion
A.  255,000   255,000
B.  270,000   280,000
C.  280,000   270,000
D.  305,000   275,000
E.  240,000   250,000

SUPPORTING CALCULATION:

Materials = 40,000 + 240,000 = 280,000
Conversion = (280,000 ­ 25,000) + .6(25,000) = 270,000

B 20. Information concerning Department A of Neeley Company for June is as follows:

Materials
Units    Costs  
Beginning work in process..................................................................... 17,000 $12,800 
Started in June........................................................................................ 82,000 69,700 
Units completed...................................................................................... 85,000
Ending work in process.......................................................................... 14,000

All materials are added at the beginning of the process. Using the average cost method, the cost per 
equivalent unit for materials is:
A. $0.825
B. $0.833
C. $0.85
D. $0.97
E. $1.01

SUPPORTING CALCULATION:  ($12,800 + $69,700) )  (85,000 + 14,000) = $.833
B 21. Kennedy Company adds materials in the beginning of the process in the Forming Department, which 
is the first of two stages of its production cycle. Information concerning the materials used in the 
Forming Department in October is as follows:

Materials
Units    Costs  
Work in process, October 1.................................................................... 6,000 $ 3,000 
Units started............................................................................................ 50,000 25,560 
Units completed and transferred out....................................................... 44,000

Using the average cost method, what was the materials cost of work in process at October 31?
A. $3,000
B. $6,120
C. $3,060
D. $5,520
E. $6,000

SUPPORTING CALCULATION:

($3,000 + $25,560) )  (44,000 + 12,000) = $.51
$.51 x 12,000 = $6,120

E 22. Roger Company manufactures Product X in a two­stage production cycle in Departments A and B. 
Materials are added at the beginning of the process in Department B. Roger uses the average costing 
method. Conversion costs for Department B were 50% complete as to the 6,000 units in beginning 
work in process and 75% complete as to the 8,000 units in ending work in process. A total of 12,000 
units were completed and transferred out of Department B during February. An analysis of the costs 
relating to work in process and production activity in Department B for February follows:

Transferred­ Materials Conversion


   in Costs       Costs        Costs   
Work in process, February 1:
Costs attached.................................................... $12,000 $2,500 $1,000
February activity:
Costs added........................................................ 29,000 5,500 5,000

The total cost per equivalent unit transferred out for February of Product X, rounded to the nearest 
penny, was:
A. $2.82
B. $2.85
C. $2.05
D. $2.75
E. $2.78

SUPPORTING CALCULATION:

Transferred­in costs = $41,000 )  20,000 = $2.05


Materials cost = $8,000 )  20,000 =  .40
Conversion cost = $6,000 )  18,000 =     .33
$2.78
A 23. Simpson Co. adds materials at the beginning of the process in Department M. The following 
information pertains to Department M's work in process during April:

Units 
Work in process on April 1
(60% complete as to conversion cost).....................................................................  3,000
Started in April............................................................................................................... 25,000
Completed in April......................................................................................................... 20,000
Work in process on April 30
(75% complete as to conversion cost).....................................................................  8,000

Under the average costing method, the equivalent units for conversion cost are:
A. 26,000
B. 25,000
C. 24,000
D. 21,800
E. none of the above

SUPPORTING CALCULATION:  20,000 + .75(8,000) = 26,000

D 24. During March, Quig Company's Department Y equivalent unit product costs, computed under the 
average cost method, were as follows:

Materials.................................................. $1
Conversion...............................................  3
Transferred­in..........................................  5

Materials are introduced at the end of the process in Department Y. There were 4,000 units (40% 
complete as to conversion costs) in work in process at March 31. The total costs assigned to the March 
31 work in process inventory should be:
A. $36,000
B. $28,800
C. $27,200
D. $24,800
E. none of the above

SUPPORTING CALCULATION:  $5(4,000) + $3(4,000 x .4) = $24,800

The following questions are based on the material in the Appendix to the chapter.

B 25. If a company reports two different unit costs for goods transferred to the next department, it is 
reasonable to assume that:
A. the department accounts for lost units at the end of the process 
B. a fifo costing method is used
C. lost unit costs are computed separately
D. an average costing method is used
E. errors must have occurred in recording costs
C 26. In order to compute equivalent units of production using the fifo method of process costing, work for 
the period must be broken down to units:
A. started and completed during the period
B. completed during the period and units in ending inventory
C. completed from beginning inventory, started and completed during the month, and units in 
ending inventory
D. started during the period and units transferred out during the period
E. processed during the period and units completed during the period

A 27. The first­in, first­out method of process costing will produce the same cost of goods manufactured 
amount as the average cost method when:
A. there is no beginning inventory
B. there is no ending inventory
C. beginning and ending inventories are each 50% complete
D. beginning inventories are 100% complete as to materials
E. goods produced are homogeneous

B 28. The fifo method of process costing differs from the average cost method of process costing in that fifo:
A. allocates costs based on whole units, but the average cost method uses equivalent units
B. considers the stage of completion of beginning work in process in computing equivalent units 
of production, but the average cost method does not
C. does not consider the stage of completion of beginning work in process in computing equivalent
units of production, but the average cost method does
D. is applicable only to those companies using the fifo inventory pricing method, but the average 
cost method may be used with any inventory pricing method
E. none of the above

A 29. Connor Company computed the flow of physical units completed for Department M for the month of 
March as follows:

Units completed:
From work in process on March 1.............................................................................. 15,000
From March production.............................................................................................. 45,000
Total..................................................................................................................... 60,000

Materials are added at the beginning of the process. The 12,000 units of work in process at March 31 
were 80% complete as to conversion costs. The work in process at March 1 was 60% complete as to 
conversion costs. Using the fifo method, the equivalent units for March conversion costs were:
A. 60,600
B. 55,200
C. 57,000
D. 54,600
E. 63,600

SUPPORTING CALCULATION:  (15,000 x .4) + 45,000 + (12,000 x .8) = 60,600
D 30. The Hilo Company computed the physical flow of units for Department A for the month of April as 
follows:

Units completed:
From work in process on April 1................................................................................ 10,000
From April production................................................................................................ 30,000
Total..................................................................................................................... 40,000

Materials are added at the beginning of the process. Units of work in process at April 30 were 8,000. 
The work in process at April 1 was 80% complete as to conversion costs, and the work in process at 
April 30 was 60% complete as to conversion costs. What are the equivalent units of production for the 
month of April using the fifo method?

Materials Conversion Costs
A. 48,000 48,000
B. 40,000 47,600
C. 36,800 38,000
D. 38,000 36,800
E. 48,000 44,800

SUPPORTING CALCULATION:

Materials = 30,000 + 8,000 = 38,000
Conversion = (10,000 x .2) + 30,000 + (8,000 x .6) = 36,800

E 31. Department A is the first stage of Mann Company's production cycle. The following information is 
available for conversion costs for the month of April:

............................................................................................................................... Units  
Beginning work in process (60% complete).................................................................... 20,000
Started in April................................................................................................................. 340,000
Completed in April and transferred to Department B...................................................... 320,000
Ending work in process (40% complete)......................................................................... 40,000

Using the fifo method, the equivalent units for the conversion cost calculation are:
A. 336,000
B. 360,000
C. 328,000
D. 320,000
E. 324,000

SUPPORTING CALCULATION:

(20,000 x .4) + 300,000 + (40,000 x .4) = 324,000

PRODUCTION LOSSES:

63
A 1. The quality costs that are associated with materials and products that fail to meet quality standards and
result in manufacturing losses are known as:
A. internal failure costs
B. external failure costs
C. prevention costs
D. appraisal costs
E. none of the above

D 2. The quality costs that are associated with designing, implementing, and maintaining the quality system
are known as:
A. appraisal costs
B. internal failure costs
C. external failure costs
D. prevention costs
E. none of the above

C 3. The quality costs that are incurred to ensure that materials and products meet quality standards are 
known as:
A. external failure costs
B. prevention costs
C. appraisal costs
D. internal failure costs
E. none of the above

B 4. The quality costs that are incurred because inferior quality products are shipped to customers are 
known as:
A. internal failure costs
B. external failure costs
C. prevention costs
D. appraisal costs
E. none of the above

64
D 5. All of the following are characteristics of total quality management except:
A. the company's objective for all business activity is to serve its customers
B. top management provides an active leadership role in quality improvement
C. all employees are actively involved in quality improvement
D. the company maintains a loosely defined system of identifying quality problems so as not to 
stifle employee creativity
E. the company provides continuous training as well as recognition for achievement

A 6. The best approach to quality improvement is to concentrate on:
A. prevention
B. detection
C. appraisal
D. increased production
E. none of the above

C 7. A mathematical technique used to monitor production quality and reduce product variability is:
A. the method of least squares
B. the statistical scattergraph method
C. statistical process control
D. linear programming
E. none of the above

D 8. Appraisal costs include all of the following except:
A. inspecting and testing materials
B. inspecting products during and after production
C. obtaining information from customers about product satisfaction
D. designing quality into the product and the production process
E. all of the above

B 9. Internal failure costs include all of the following except:
A. the cost of the scrap
B. the cost of warranty repairs and replacements
C. rework
D. downtime due to machine failures
E. all of the above

E 10. All of the following accounts would be acceptable ones to credit at the time scrap is sold except:
A. Scrap Sales
B. Cost of Goods Sold
C. Factory Overhead Control
D. Work in Process
E. all of the above would be acceptable

C 11. Scrap includes all of the following except:
A. the trimmings remaining after processing materials
B. defective materials that cannot be used or returned to the vendor
C. partially or fully completed units that are in some way defective
D. broken parts resulting from employee or machine failures
E. all of the above
A 12. When spoilage occurs because of some action taken by the customer, the unrecoverable cost of the 
spoilage should be charged to:
A. Work in Process
B. Spoiled Goods Inventory
C. Factory Overhead Control
D. Applied Factory Overhead
E. none of the above

C 13. When spoilage occurs because of some internal failure, the unrecoverable cost should be charged to:
A. Work in Process
B. Spoiled Goods Inventory
C. Factory Overhead Control
D. Applied Factory Overhead
E. none of the above

A 14. When rework occurs because of some action taken by the customer, the cost of the rework should be 
charged to:
A. Work in Process
B. Spoiled Goods Inventory
C. Factory Overhead Control
D. Applied Factory Overhead
E. none of the above

C 15. When rework occurs because of some internal failure, the cost of the rework should be charged to:
A. Work in Process
B. Spoiled Goods Inventory
C. Factory Overhead Control
D. Applied Factory Overhead
E. none of the above

C 16. Newman Company's Job 1865 for the manufacture of 2,200 coats was completed during August at the 
unit costs presented below.  Due to an internal failure in the production process, 200 coats were found 
to be spoiled during final inspection that were sold to a jobber for $6,000. 

Direct materials.................................................................................................................. $20
Direct labor........................................................................................................................ 18
Factory overhead................................................................................................................  18
................................................................................................................................ $56

What would be the unit cost of good coats produced on Job 1865?
A. $57.00
B. $55.00
C. $56.00
D. $58.00
E. none of the above

SUPPORTING CALCULATION:  $20 + $18 + $18 = $56
A 17. During March, Vaughan Company incurred the following costs on Job 009 for the manufacture of 200 
motors:

Original cost accumulation:
Direct materials........................................................................................................... $ 660
Direct labor................................................................................................................. 800
Factory overhead (150% of direct labor)....................................................................    1,200
     
 $   
   2,660

Direct costs of reworking 10 units:
Direct materials........................................................................................................... $100
Direct labor.................................................................................................................  160
..................................................................................................................................... $260

The rework costs were attributable to the exacting specifications of the customer. What is the cost per 
finished unit of Job 009?
A. $15.80
B. $14.60
C. $14.00
D. $13.30
E. none of the above

SUPPORTING CALCULATION:  

$2,660 + $260 + (150% x $160) = $3,160 )  200 = $15.80

C 18. Spoilage occurs as a result of an internal failure in a process cost system.  Using average costing, the 
number of equivalent units that production costs should be charged to would be based upon:
A. spoiled units
B. units transferred out and spoiled units
C. units transferred out, spoiled units, and units in ending inventory
D. units transferred out and units in ending inventory
E. none of the above

D 19. Spoilage occurs as a result of normal production shrinkage in a process cost system.  Using average 
costing, the number of equivalent units that production costs should be charged to would be based 
upon:
A. spoiled units
B. units transferred out and spoiled units
C. units transferred out, spoiled units, and units in ending inventory
D. units transferred out and units in ending inventory
E. none of the above

C 20. In a process cost system, the cost of spoilage due to an internal production failure should be recorded 
as: 
A. dr. Work in Process; cr. Finished Goods
B. dr. Work in Process; cr. Factory Overhead Control 
C. dr. Factory Overhead Control; cr. Work in Process 
D. dr. Materials; cr. Factory Overhead
E. dr. Finished Goods; cr. Work in Process
B 21. Gyro Products transferred 10,000 units to one department.  An additional 3,000 units of materials were
added in the department.  At the end of the month, 7,000 units were transferred to finished goods; 
while 4,000 units remained in work in process inventory.  There was no beginning inventory, and lost 
units were a result of normal production shrinkage.  The production costs for the period in this 
department would be effectively allocated over: 
A. 12,000 units
B. 11,000 units
C. 10,000 units
D. 7,000 units
E. 13,000 units

SUPPORTING CALCULATION:  7,000 + 4,000 = 11,000

B 22. In manufacturing its products for the month of March, Leo Co. incurred normal production shrinkage 
of $10,000 and spoilage due to internal failure of $12,000. How much spoilage cost should Leo charge 
to Factory Overhead Control for the month of March? 
A. $22,000
B. $12,000
C. $10,000
D. $0
E. none of the above

C 23. Willis, Inc. instituted a new process in October.  During October, 10,000 units were started in 
Department A.  Of the units started, 1,000 were lost in the process due to normal production shrinkage,
7,000 were transferred to Department B, and 2,000 remained in work in process at October 31.  The 
work in process at October 31 was 100% complete as to materials costs and 50% complete as to 
conversion costs.  Materials costs of $27,000 and conversion costs of $40,000 were charged to 
Department A in October.  What were the total costs transferred to Department B? 
A. $46,900
B. $53,600
C. $56,000
D. $57,120
E. none of the above

SUPPORTING CALCULATION:

Materials:  $27,000 )  (7,000 + 2,000) = $3
Conversion:  $40,000 )  (7,000 + 1,000) = $5
Transferred costs:  7,000 x $8 = $56,000
D 24. A company that manufactures baseballs begins operations on January 1.  Each baseball requires three 
elements: a hard plastic core, several yards of twine that are wrapped around the plastic core, and a 
piece of leather to cover the baseball.  The plastic core is started down a conveyor belt and is 
automatically wrapped with the twine to the approximate size of the baseball, at which time the leather
cover is sewn to the wrapped twine.  Finished baseballs are inspected, and the ones that are defective 
due to internal production failure are pulled out.  Defective baseballs cannot be economically salvaged
and are destroyed.  Cost and production reports for the first week of operations are: 

Raw material cost................................................................................................................. $ 840
Conversion cost....................................................................................................................          315
  
...................................................................................................................................  $      1,155
 

During the week, 2,100 baseballs were completed; 2,000 passed inspection.  There was no ending 
work in process.  The cost of the spoilage charged to Factory Overhead is:
A. $33
B. $22
C. $1,100
D. $55
E. none of the above

SUPPORTING CALCULATION:

Materials:  $840 )  (2,000 + 100) = $.40
Conversion:  $315 )  (2,000 + 100) = $.15
Spoilage:  100 x $.55 = $55

A 25. In a process cost system, the cost of rework usually is debited to:
A. Factory Overhead Control
B. Applied Factory Overhead
C. Spoiled Goods Inventory
D. Work in Process
E. none of the above

The following questions are based on the Appendix to the chapter:

D 26. If spoilage occurs as a result of an internal failure in a process cost system, using fifo costing, the 
number of equivalent units that production costs should be charged to would be based upon:
A. spoiled units
B. units transferred out and spoiled units
C. units transferred out, beginning inventory, and units in ending inventory
D. units transferred out, spoiled units, units in ending inventory, and units in beginning inventory
E. none of the above
C 27. If spoilage occurs as a result of normal production shrinkage in a process cost system, using fifo 
costing, the number of equivalent units that production costs should be charged to would be based on:
A. spoiled units
B. units transferred out and spoiled units
C. units transferred out, beginning inventory, and units in ending inventory
D. units transferred out, spoiled units, units in ending inventory, and units in beginning inventory
E. none of the above

B 28. Primo Products transferred 15,000 units to one department.  An additional 5,000 units were in 
beginning inventory in the department.  At the end of the month, 12,000 units were transferred to the 
next department, 6,000 units remained in work in process, 40% complete as to conversion costs and 
the remaining units were lost at the 75% stage of conversion.  Beginning inventory was 60% complete 
as to conversion costs and lost units were the result of internal failure.  The equivalent units of 
conversion cost using fifo costing is:
A. 14,400
B. 12,900
C. 13,900
D. 13,400
E. none of the above

SUPPORTING CALCULATION:

Equivalent units in beginning inventory (40% x 5,000).................................................... 2,000
Equivalent units started and completed during period 
(12,000 ­ 5,000)......................................................................................................... 7,000
Equivalent units in ending inventory (40% x 6,000)......................................................... 2,400
Equivalent units of spoilage (75% x 2,000).......................................................................  1,500
Total equivalent units......................................................................................................... 12,900

A 29. Primo Products transferred 15,000 units to one department.  An additional 5,000 units were added in 
the department.  At the end of the month, 12,000 units were transferred to the next department, 6,000 
units remained in work in process, 40% complete as to conversion costs and the remaining units were 
lost at the 75% stage of conversion.  Beginning inventory was 60% complete as to conversion costs, 
and lost units were the result of normal production shrinkage.  The equivalent units of conversion cost 
using fifo is:
A. 11,400
B. 14,400
C. 12,900
D. 13,400
E. none of the above

SUPPORTING CALCULATION:

Equivalent units in beginning inventory (40% x 5,000).................................................... 2,000
Equivalent units started and completed during period 
(12,000 ­ 5,000)......................................................................................................... 7,000
Equivalent units in ending inventory (40% x 6,000).........................................................   2,400
Total equivalent units......................................................................................................... 11,400
JOINT PRODUCTS:

B 1. The allocation of joint costs to individual products is useful primarily for purposes of:
A. determining whether to produce one of the joint products 
B. inventory costing
C. determining the best market price
D. deciding whether to sell at the split­off point
E. evaluating whether an output is a main product or a by­product

B 2. The method used for the allocation of joint costs to products is important:
A. only in the minds of accountants
B. because profits will be affected when ending inventories change from the beginning of the 
period
C. because its validity for justifying prices before regulatory authorities is unquestioned
D. because profit margins differ when the relative sales value method is used
E. for income determination when inventories are nonexistent

A 3. In a joint production process, a by­product is also described as:
A. a simultaneously produced product of relatively low value 
B. a form of main product with controllable production proportions 
C. waste
D. products of low value recovered at the end of a production process
E. a product with no value contribution to help offset production costs

D 4. All of the following are methods of costing by­products except the:
A. market value method
B. recognition of net revenue method
C. recognition of gross revenue method
D. average unit cost method
E. replacement cost method

71
72 Chapter 8

E 5. Reporting revenue from by­product sales on the income statement as additional sales revenue:
A. allocates costs to by­products on the basis of quantities produced
B. reduces the main product cost by the estimated market value of the by­product
C. credits main product costs only when the by­product is used in further production
D. allocates a proper share of production costs to the by­product 
E. overstates ending inventory costs of the main product

E 6. All of the following are methods of allocating joint production costs except the:
A. market value method
B. quantitative unit method
C. average unit cost method
D. average cost method
E. recognition of net revenue method

D 7. Tobin Company manufactures products S and T from a joint process. The market value at split­off was
$50,000 for 6,000 units of Product S and $50,000 for 2,000 units of Product T. Assuming that the 
portion of the total joint cost properly allocated to Product S using the market value method was 
$30,000, the total joint cost was: 
A. $40,000
B. $42,500
C. $45,000
D. $60,000
E. $75,000

SUPPORTING CALCULATION:

C 8. Costs to be incurred after the split­off point are most useful for:
A. adjusting inequities in the joint cost allocation procedure
B. determining the levels of joint production
C. assessing the desirability of further processing
D. setting the mix of output products
E. assessing sales realization values for allocating joint costs accurately
Costing By­Products and Joint Products 73

D 9. Alphabet Company manufactures Products A and B from a joint process that also yields a by­product, 
X. Alphabet accounts for the revenues from its by­product sales as a deduction from the cost of goods 
sold of its main products. Additional information is as follows:

A B X Total
Units produced................................ 15,000 9,000 6,000 30,000
Joint costs........................................ $ 264,000
Market value at split­off................. $290,000 $150,000 $ 10,000 $450,000

Assuming that joint product costs are allocated using the market value at the split­off approach, the 
joint cost allocated to Product B would be:
A. $136,540
B. $79,200
C. $88,000
D. $86,591
E. $99,000

SUPPORTING CALCULATION:

D 10. If a company obtains two salable products from the refining of one ore, the refining process should be 
accounted for as a(n):
A. reduction process
B. depletion process
C. mixed cost process
D. joint process
E. extractive process

A 11. The assignment of raw material costs to the major end products resulting from refining a barrel of 
crude oil is best described as:
A. joint costing
B. differential costing
C. incremental costing
D. variable costing
E. indirect costing

B 12. The following components of production that can be allocated as joint costs when a single 
manufacturing process produces several salable products are:
A. indirect production costs only
B. materials, labor, and overhead
C. materials and labor only
D. labor and overhead only
E. overhead and materials only
Costing By­Products and Joint Products 74

A 13. The following statement that best describes a by­product is:
A. a product that usually produces a small amount of revenue when compared to the main 
product's revenue
B. a product that does not bear any portion of the joint processing costs
C. a product that is produced from material that would otherwise be scrap
D. a product that has a lower unit selling price than the main product
E. a product created along with the main product whose sales value does not cover its cost of 
production

B 14. Relative sales value at split­off is used to allocate:
Cost Beyond
   Split­Off    Joint Costs
A. yes no
B. no yes
C. no no
D. sometimes never
E. yes yes

B 15. The following is acceptable regarding the allocation of joint product costs to a by­product:

None Allocated Some Portion Allocated
A. not acceptable not acceptable
B. acceptable acceptable
C. acceptable not acceptable
D. sometimes acceptable never acceptable
E. not acceptable acceptable

D 16. Idaho Corporation manufactures liquid chemicals A and B from a joint process. Joint costs are 
allocated on the basis of relative market value at split­off. It costs $4,560 to process 500 gallons of 
Product A and 1,000 gallons of Product B to the split­off point. The market value at split­off is $10 per
gallon for Product A and $14 for Product B. Product B requires an additional process beyond split­off 
at a cost of $2 per gallon before it can be sold. What is Idaho's cost to produce 1,000 gallons of 
Product B?
A. $5,040
B. $4,360
C. $4,860
D. $5,360
E. $3,360

SUPPORTING CALCULATION:
Costing By­Products and Joint Products 75

C 17. Harry Corp. manufactures Products J, K, L, and M from a joint process. Additional information is as 
follows:

Market       If Processed Further    
Units Value at Additional Market
Product Produced  Split­Off     Costs     Value 
J 6,000 $ 80,000 $ 7,500 $ 90,000
K 5,000 60,000 6,000 70,000
L 4,000 40,000 4,000 50,000
M  3,000        20,000
            2,500
            30,000
    
18,000  $   
    200,000  $       20,000  $   
    240,000

Assuming that total joint costs of $160,000 were allocated using the market value at split­off approach,
what joint costs were allocated to each product?

J K L M
A. $53,333 $44,444 $35,556 $26,667
B. $60,000 $46,667 $33,333 $20,000
C. $64,000 $48,000 $32,000 $16,000
D. $60,000 $48,000 $32,000 $20,000
E. $40,000 $40,000 $40,000 $40,000

SUPPORTING CALCULATION:

J: 40% x $160,000 = $64,000
K: 30% x $160,000 = $48,000
L: 20% x $160,000 = $32,000
M: 10% x $160,000 = $16,000

E 18. Cayan Company manufactures three main products, F, G, and W, from a joint process. Joint costs are 
allocated on the basis of relative market value at split­off. Additional information for June production 
activity follows:

F G W Total
Units produced.................................. 50,000 40,000 10,000 100,000
Joint costs......................................... ? ? ? $450,000
Market value at split­off................... $ 420,000 $ 270,000 $60,000 $750,000
Additional costs if 
processed further....................... $ 88,000 $ 30,000 $12,000 $130,000
Market value if 
processed further....................... $ 538,000 $ 320,000 $87,000 $945,000

Assuming that the 10,000 units of W were processed further and sold for $87,000, what was Cayan's 
gross profit on this sale? 
A. $75,000
B. $51,000
C. $21,000
D. $28,500
E. $39,000
Costing By­Products and Joint Products 76

SUPPORTING CALCULATION:

Sales:........................................................................................................ $87,000
Cost of Goods Sold:
Joint Costs......................................................................................... $36,000
Separable Costs.................................................................................  12,000  48,000
Gross Profit.............................................................................................. $39,000

B 19. A company manufactures two joint products at a joint cost of $1,000. These products can be sold at 
split­off, or when further processed at an additional cost, sold as higher quality items. The decision to 
sell at split­off or further process should be based on the: 
A. allocation of the $1,000 joint cost using the quantitative unit measure
B. assumption that the $1,000 joint cost is irrelevant
C. allocation of the $1,000 joint cost using the relative sales value approach
D. assumption that the $1,000 joint cost must be allocated using a physical­measure approach
E. allocation of the $1,000 joint cost using any equitable and rational allocation basis

D 20. The characteristic that is most often used to distinguish a product as either a joint product or a by­
product is the:
A. amount of labor used in processing the product
B. amount of separable product costs that are incurred in processing 
C. amount (i.e., weight, inches, etc.) of the product produced in the manufacturing process
D. relative sales value of the products produced in the process
E. none of the above

A 21. A company processes raw material into products F1, F2, and F3. Each ton of raw material produces 
five units of F1, two units of F2, and three units of F3. Joint processing costs to the split­off point are 
$15 per ton. Further processing results in the following per unit figures:

F1 F2 F3
Additional processing costs per unit............................ $28 $30 $25
Selling price per unit....................................................  30  35  35

If joint costs are allocated by the net realizable value of finished product, what proportion of joint 
costs should be allocated to F1? 
A. 20%
B. 30%
C. 33 1/3%
D. 50%
E. none of the above

SUPPORTING CALCULATION:
Costing By­Products and Joint Products 77

B 22. Jeffrey Co. manufactures Products A and B from a joint process. Market value at split­off was 
$700,000 for 10,000 units of A, and $300,000 for 15,000 units of B. Using the market value at split­off
approach, joint costs properly allocated to A were $140,000. Total joint costs were:
A. $98,000
B. $200,000
C. $233,333
D. $350,000
E. none of the above

SUPPORTING CALCULATION:

C 23. A company produces three main joint products and one by­product. The by­product's relative market 
value is quite low compared to that of the main products. The preferable accounting for the by­
product's net realizable value is as:
A. an addition to the revenues of the other products allocated on their respective net realizable 
values
B. revenue in the period in which it is sold
C. a reduction in the joint cost to be allocated to the three main products
D. a separate net realizable value upon which to allocate some of the joint costs
E. none of the above

C 24. A company manufactures Products X and Y using a joint process. The joint processing costs are 
$10,000. Products X and Y can be sold at split­off for $12,000 and $8,000 respectively. After split­off, 
Product X is processed further at a cost of $5,000 and sold for $21,000, whereas Product Y is sold 
without further processing. If the company uses the market value method for allocating joint costs, the 
joint cost allocated to X is:
A. $4,000
B. $5,000
C. $6,000
D. $6,667
E. none of the above

SUPPORTING CALCULATION:
Costing By­Products and Joint Products 78

D 25. The Hovart Corporation manufactures two products out of a joint processCCompod and Ultrasene. 
The joint (common) costs incurred are $250,000 for a standard production run that generates 120,000 
gallons of Compod and 80,000 gallons of Ultrasene. Compod sells for $2.00 per gallon, while 
Ultrasene sells for $3.25 per gallon. If there are no additional processing costs incurred after the split­
off point, the amount of joint cost of each production run allocated to Compod by the quantitative unit 
method is:
A. $100,000
B. $120,000
C. $130,000
D. $150,000
E. some amount other than those given above

SUPPORTING CALCULATION:

 
A 26. Ace Company produced 20,000 units of Clubs, 15,000 units of Diamonds, and 10,000 units of Hearts.  
If the company uses the average unit cost method of allocating joint production costs, which were 
$120,000 for the period, the joint costs allocated to Diamonds would be:
A. $40,000
B. $20,000
C. $80,000
D. $45,000
E. none of the above

SUPPORTING CALCULATION:

C 27. A company uses the weighted average method to assign joint products.  Weight factors used to assign 
joint costs to its three joint products were:  Product A, 4 points; Product B, 7 points; and Product C, 8 
points.  Units produced were:  Product A, 10,000; Product B, 5,000; and Product C, 3,125.  The 
amount of the joint costs of $100,000 that would be allocated to Product C are:
A. $42,105
B. $17,241
C. $25,000
D. $30,000
E. none of the above

SUPPORTING CALCULATION:
Costing By­Products and Joint Products 79

E 28. The two standards in the Standards of Ethical Conduct for Management Accountants that pertain most 
specifically to consideration of joint costs allocation are:
A. competence and confidentiality
B. confidentiality and integrity
C. competence and integrity
D. confidentiality and objectivity
E. none of the above
Costing By­Products and Joint Products 80

MATERIALS:

A 1. The cycle of materials procurement and use includes all of the following steps except for:
A. determining the cost of goods sold
B. the production budget
C. preparing the receiving report
D. maintaining the materials ledger
E. engineering to determine materials specifications

E 2. In a well­controlled materials system, the Purchasing Department performs all of the following 
activities except the:
A. placing of purchase orders with suppliers
B. receiving of purchase requisitions
C. maintaining of information on market prices for goods used 
D. preparation of purchase orders
E. approving and checking of invoices

B 3. The purchase requisition is a document used to:
A. initiate the return of merchandise to the vendor
B. inform the purchasing agent of a need for a materials item 
C. initiate payment for merchandise received
D. inform the Purchasing Department of a receipt of goods 
E. authorize the vendor to supply merchandise or materials

B 4. The expense that theoretically is not a correct part of inventory cost is:
A. freight­in
B. freight­out
C. inspection costs
D. accounting costs for materials received
E. purchasing costs

C 5. Theoretically, cash discounts permitted on purchased raw materials should be:
A. added to other income, whether taken or not
B. added to other income, only if taken
C. deducted from inventory, whether taken or not
D. deducted from inventory, only if taken
E. none of the above

80
Materials:  Controlling, Costing, and Planning 81

E 6. The materials requisition:
A. is the list of materials requirements for each step in the production sequence
B. informs the purchasing agent of the quantity and kind of materials needed
C. contracts for quantities to be delivered
D. certifies quantities received and reports results of inspection and testing
E. authorizes the storeroom to deliver types and quantities of materials to a given department

C 7. The purchase order:
A. is the list of materials requirements for each step in the production sequence
B. informs the purchasing agent of the quantity and kind of materials needed
C. contracts for quantities to be delivered
D. certifies quantities received and reports results of inspection and testing
E. authorizes the storeroom to deliver types and quantities of materials to a given department

A 8. The bill of materials:
A. is the list of materials requirements for each step in the production sequence
B. informs the purchasing agent of the quantity and kind of materials needed
C. contracts for quantities to be delivered
D. certifies quantities received and reports results of inspection and testing
E. authorizes the storeroom to deliver types and quantities of materials to a given department

D 9. The receiving report:
A. is the list of materials requirements for each step in the production sequence
B. informs the purchasing agent of the quantity and kind of materials needed
C. contracts for quantities to be delivered
D. certifies quantities received and reports results of inspection and testing
E. authorizes the storeroom to deliver types and quantities of materials to a given department

D 10. The purchasing department performs all of the following functions except:
A. receives purchase requisitions for materials, supplies, and equipment
B. keeps informed concerning sources of supply, prices, and delivery schedules
C. prepares and places purchase orders
D. compares quantities received with the suppliers' packing list
E. arranges for the reporting among the purchasing, receiving, and accounting departments

C 11. The purchase requisition may originate with all of the following except:
A. a storeroom employee
B. a materials record clerk
C. a receiving department clerk
D. a research, engineering, or other department employee who needs materials of a special nature
E. a computer

B 12. The receiving department does all of the following except:
A. unloads and unpacks incoming materials
B. keeps informed concerning sources of supply, prices, and delivery schedules
C. matches materials received with descriptions on purchase orders
D. arranges for inspection, when necessary
E. routes accepted materials to the appropriate departments
Materials:  Controlling, Costing, and Planning 82

A 13. A cost of having too few items on hand in inventory is:
A. frequent stockouts
B. excessive insurance costs
C. payment of additional warehouse space
D. spoilage costs
E. costs of obsolescence

B 14. Of the following, the expense that is not relevant to determining the most economic quantity to order 
is:
A. additional costs to store inventory
B. rental of warehouse space under a ten­year lease
C. interest expense of financing purchases
D. spoilage costs
E. variable costs of placing an order

B 15. A company has been ordering more than the economic order quantity.  This would result in:
A. more frequent order points
B. carrying costs greater than order costs
C. equal safety stock costs and carrying costs
D. carrying costs less than order costs
E. insufficient safety stock costs

B 16. Annual demand for squash racquets is 50,000 units, and carrying costs amount to $2 per unit.  Order 
costs for the company amount to $5.  The optimum order quantity in units for squash racquets is 
(rounded to the nearest unit):
A. 191
B. 500
C. 250
D. 100
E. 625

SUPPORTING CALCULATION:

E 17. A company orders 10,000 units (a one­year supply) of Zap at one time.  Zap costs $1 per unit, and 
order costs amount to $500 each time an order is placed.  The costs to carry Zap in inventory amount 
to 20% of the materials cost.  For an entire year, the inventory carrying costs and order costs are:
A. $2,000
B. $200
C. $500
D. $1,000
E. $1,500

SUPPORTING CALCULATION:
Materials:  Controlling, Costing, and Planning 83

B 18. If the average lead time and usage figures are used for determining the order point, then the probability
of a stockout is:
A. .005%
B. 50%
C. 5%
D. 100%
E. 2.5%

A 19. There are 1,000 Trolls in stock, and 1,500 are due in from orders that were placed previously.  The 
company sells Trolls at the rate of 100 per day and finds that it takes an average of 20 days for an 
order to be received.  Because usage and lead times are known with certainty and because the 
company has determined that an order must be placed now, the desired safety stock quantity must be 
equal to:
A. 500 units
B. 1,000 units
C. 2,500 units
D. 100 units
E. 1,500 units

SUPPORTING CALCULATION:

1,000 + 1,500 = (100 x 20) + SSQ

SSQ = 500

B 20. The use of quantitative models can be modified to improve the management of inventory by:
A. including only fixed costs in the EOQ analysis
B. employing a minimum safety stock level because delivery time and inventory usage rates may 
vary
C. purchasing inventory only once a year to save on ordering cost
D. purchasing inventory monthly to save on carrying cost
E. eliminating semivariable costs from any consideration in the EOQ analysis because of the 
difficulty of estimating those costs

C 21. An inventory control technique that reviews quantities on hand periodically and orders sufficient 
quantities to bring inventory up to a desired level expressed as a number of days' or weeks' supply is 
the:
A. two­bin method
B. ABC inventory control method
C. order cycling method
D. min­max method
E. automatic order point system

B 22. The factor that need not be considered when calculating an inventory economic order quantity (EOQ) 
is:
A. annual sales of a product
B. safety stock level
C. order­placing costs
D. storage costs
E. risk of inventory obsolescence and deterioration
B 23. Brad Company has correctly computed its economic order quantity as 500 units. However, 
management would rather order in quantities of 600 units.  How will Brad's total annual purchase 
order cost and total annual carrying cost for an order quantity of 600 units compare to the respective 
amounts for an order quantity of 500 units?
A. higher purchase order cost and lower carrying cost
B. lower purchase order cost and higher carrying cost
C. higher purchase order cost and higher carrying cost
D. lower purchase order cost and lower carrying cost
E. none of the above

A 24. Carter Company buys a certain part for its manufacturing process for $20 a part and needs 10,000 parts
a year.  It costs $3 a year to carry one of these parts in inventory.  The cost of placing a purchase order 
for these parts is $15.  Assuming that the parts will be required evenly throughout the year, the formula

for the economic order quantity is the square root of:
A 25. For its economic order quantity model, a company has a $10 cost of placing an order and a $2 annual 
cost of carrying one unit in stock.  If the cost of placing an order increases by 20%, the annual cost of 
carrying one unit in stock increases by 25%, and all other considerations remain constant, the 
economic order quantity will:
A. decrease
B. increase
C. remain unchanged
D. either increase or decrease, depending on the reorder point
E. either increase or decrease, depending on the safety stock

C 26. For inventory management, ignoring safety stocks, a valid computation of the reorder point is:
A. order costs plus carrying costs
B. the square root of the anticipated demand during lead time
C. the anticipated demand per day during lead time times lead time in days
D. the economic order quantity
E. the economic order quantity times the anticipated demand during lead time
C 27. The Cappalari Company wishes to determine the amount of safety stock that it should maintain for 
Product D to result in the lowest cost.  The following information is available:

Stockout cost................................................................................................ $ 80 per occurrence
Carrying cost of safety stock....................................................................... $ 2 per unit
Number of purchase orders.......................................................................... 5 per year

The options available to Cappalari are as follows:

Units of Probability of Running
Safety Stock out of Safety Stock
10 50%
30 30%
50 10%
55 5%

The number of units of safety stock that will result in the lowest cost is:
A. 30
B. 50
C. 55
D. 10
E. none of the above

SUPPORTING CALCULATION:

Safety Expected Stockout Carrying Stockout and


Stock Stockouts    Cost       Cost    Carrying Cost
10 2.5 $200 $  20 $220
30 1.5 120 60 180
50 .5 40 100 140
55 .25 20 110 130

B 28. The following information is available for Odyssey Company's Material Y:

Annual usage in units........................................................................................................... 10,000
Working days per year......................................................................................................... 250
Normal lead time in working days....................................................................................... 30
Maximum lead time in working days.................................................................................. 70

Assuming that the units of Material Y will be required evenly throughout the year, the order point 
would be:
A. 2,000
B. 2,800
C. 2,105
D. 1,200
E. 1,600

SUPPORTING CALCULATION:

[(10,000 )  250) x 30] + [(70 ­ 30) x 40]
30 (10,000 )  250) + 40 (70 ­ 30) = 2,800
A 29. The following information relates to Hudson Company's Material A:

Annual usage in units........................................................................................................... 7,200
Working days per year......................................................................................................... 240
Normal lead time in working days....................................................................................... 20
Maximum lead time in working days.................................................................................. 45

Assuming that the units of Material A will be required evenly throughout the year, the safety stock and
order point would be:

Safety Stock Order Point
A. 750 1,350
B. 600 750
C. 600 1,350
D. 750 600
E. none of the above

SUPPORTING CALCULATION:

Safety Stock:  (7,200 )  240) (45 ­ 20) = 750
Order Point:  20 (7,200 )  240) + 750 = 1,350

C 30. Penguin Company manufactures winter jackets.  Setup costs are $2.00.  Penguin manufactures 4,000 
jackets evenly throughout the year.  Using the economic order quantity approach, the optimal 
production run would be 200 when the cost of carrying one jacket in inventory for one year is:
A. $0.10
B. $0.20
C. $0.40
D. $0.05
E. none of the above

SUPPORTING CALCULATION:
A 31. The following data refer to various annual costs relating to the inventory of a single­product company:

Unit transportation­in on purchases..................................................................................... $ .20
Storage per unit.................................................................................................................... .12
Insurance per unit................................................................................................................. 10
Annual interest foregone from alternate investment of funds............................................. $ 800
Annual number of units required......................................................................................... 10,000

What is the annual carrying cost per unit?
A. $.30
B. $.42
C. $.50
D. $.32
E. $.22

SUPPORTING CALCULATION:

D 32. Bliss Company has an order point at 1,400 units, usage during normal lead time of 600 units, and an 
EOQ of 2,000 units.  Its maximum inventory, assuming normal lead time and usage, would be:
A. 3,400 units
B. 2,000 units
C. 1,200 units
D. 2,800 units
E. 4,000 units

SUPPORTING CALCULATION:

(1,400 ­ 600) + 2,000 = 2,800

A 33. The inventory model that follows the concept that 80% of the value of an inventory is in 20% of the 
inventory items is the:
A. ABC plan
B. economic order quantity (EOQ) model
C. just­in­time inventory system
D. materials requirements planning (MRP) system
E. zero inventory model

B 34. The materials control method that is based on the premise that the quantities of most stock items are 
subject to definable limits is the:
A. cycle review method
B. min­max method
C. two­bin method
D. ABC plan
E. none of the above
C 35. The materials control method that is based on physical observation that an order point has been 
reached is the:
A. cycle review method
B. min­max method
C. two­bin method
D. ABC plan
E. none of the above

The following questions are based on the Appendix to the chapter:

C 36. If the cost of goods sold computed when inventory is costed using the fifo method is less than when 
using the lifo method:
A. prices decreased
B. prices remained unchanged
C. prices increased
D. price trend cannot be determined from the information given 
E. prices went up and down

A 37. The method of inventory pricing that best approximates specific identification of the actual flow of 
costs and units in most manufacturing situations is:
A. first­in, first­out
B. last­in, first­out
C. base stock
D. average cost
E. none of the above

D 38. The following information was available from the inventory records of the Anthony Company for 
January 19X7:

Unit Total
Units Cost Cost
Balance at January 1, 19X7......................................... 2,000 $  9.775 $19,550
Purchases:
January 6, 19X7.................................................... 1,500 10.300 15,450
January 26, 19X7.................................................. 3,400 10.750 36,550
Sales:
January 7, 19X7.................................................... 1,800
January 31, 19X7.................................................. 3,200
Balance at January 31, 19X7........................................ 1,900

Assuming that Anthony maintains perpetual inventory records, what should be the inventory at 
January 31, 19X7, using the average cost inventory method rounded to the nearest dollar?
A. $19,998
B. $19,523
C. $19,703
D. $19,950
E. none of the above

SUPPORTING CALCULATION:
D 39. The following information was available from the inventory records of the Anthony Company for 
January 19X7:

Unit Total
Units Cost Cost
Balance at January 1, 19X7......................................... 2,000 $  9.775 $19,550
Purchases:
January 6, 19X7.................................................... 1,500 10.300 15,450
January 26, 19X7.................................................. 3,400 10.750 36,550
Sales:
January 7, 19X7.................................................... 1,800
January 31, 19X7.................................................. 3,200
Balance at January 31, 19X7........................................ 1,900

Assuming that Anthony does not maintain perpetual inventory records, what should be the inventory at
January 31, 19X7, using the average cost inventory method rounded to the nearest dollar?
A. $19,950
B. $19,998
C. $19,523
D. $19,702
E. none of the above

SUPPORTING CALCULATION:

A 40. In a period of rising prices, using which of the following inventory cost flow methods would result in 
the highest ending inventory?
A. fifo
B. average cost
C. weighted average cost
D. moving average cost
E. lifo

A 41. The inventory cost flow method that involves computations based on broad inventory pools of similar 
items is:
A. dollar­value lifo
B. average cost
C. moving average
D. fifo
E. regular quantity of goods lifo

BACKFLUSH/JIT:

D 1. One of the requirements for a JIT system to be successful is:
A. cyclical production
B. adequate inventory stock
C. coupling it with job order costing
D. high quality and balanced work loads
E. all of the above

89
A 2. All of the following are terms used to describe the JIT effort to reduce inventories of work in process 
and raw materials, except:
A. backflush production
B. stockless production
C. lean production
D. ZIP production
E. none of the above are appropriate terms

C 3. The JIT production ideal is a batch size of:
A. one hundred
B. ten
C. one
D. zero
E. none of the above

C 4. The objective of reducing inventory to zero is possible if all of the following conditions are present, 
except:
A. low or insignificant setup costs
B. minimum lead times
C. long setup times
D. balanced and level work loads
E. no interruptions due to stockouts

D 5. The continuing reduction of inventories is achieved by all of the following steps except:
A. inventories are reduced until a problem is discovered
B. once the problem is defined the inventory level is increased to keep the system operating 
smoothly
C. the problem is analyzed and practical ways are identified to reduce it
D. once the problem is removed, the inventory level is increased until another problem is 
discovered
E. all of the above steps are required

90
Just­in­Time and Backflushing 91

B 6. In a JIT system, velocity is inversely related to:
A. backflushing
B. throughput time
C. acceleration
D. zero inventory production
E. none of the above

D 7. If 500 units are produced per day and 2,000 units are in process at any time, the throughput time is:
A. 1/2 day
B. 1/4 day
C. two days
D. four days
E. none of the above

SUPPORTING CALCULATION:

D 8. In a JIT system, if the rate of output is doubled while the number of units in process is cut in half, then 
the speed of the system has been:
A. reduced by 25%
B. doubled
C. reduced by 50%
D. quadrupled
E. none of the above

A 9. Of the following, the only activity that adds value to a product is:
A. processing time
B. moving time
C. waiting time
D. inspection time
E. all of the above

B 10. If the annual carrying cost percentage is 30% and average work in process is $300,000 and 
management plans to use JIT to double the velocity of work in process without changing total annual 
output, the savings in annual carrying costs will be:
A. $90,000
B. $45,000
C. $150,000
D. $180,000
E. none of the above

SUPPORTING CALCULATION:

30% x 1/2 x $300,000 = $45,000
Just­in­Time and Backflushing 92

A 11. If Step 1 in a production process processes each unit and sends it to await Step 2, and 500 units are 
waiting between Steps 1 and 2, how many defective units might Step 1 produce before the problem is 
detected in Step 2?
A. 500
B. an unlimited number
C. 250
D. 1,000
E. none of the above

C 12. Assume that a company plans a reduction in work in process levels of 50% and has an annual 
inventory carrying cost of 20% and a past average cost of work in process of $75,000.  The 50% 
reduction in work in process would be expected to produce annual savings of:
A. $37,500
B. $15,000
C. $7,500
D. $3,750
E. none of the above

SUPPORTING CALCULATION:

50% x 20% x $75,000 = $7,500

E 13. Alpha Company has 10 work stations where work in process is held, 100 average units in work in 
process per station, an average cost of a unit in work in process of $75, and an annual inventory 
carrying cost of 20%.  If Alpha plans a 50% reduction in work in process levels, the expected annual 
savings in carrying costs would be:
A. $37,500
B. $15,000
C. $30,000
D. $3,750
E. none of the above

SUPPORTING CALCULATION:

50% x 20% x (10 x 100 x $75) = $7,500

B 14. Beta Company has an average dollar loss per defective unit of $25, a planned reduction in number of 
defective units produced per out­of­control condition of 5, and the number of out­of­control conditions
not discovered immediately is 250.  The expected savings in cost of defects would be:
A. $1,250
B. $31,250
C. $6,250
D. $125
E. none of the above

SUPPORTING CALCULATION:

$25 x 5 x 250 = $31,250
Just­in­Time and Backflushing 93

B 15. Beta Company has an average dollar loss per defective unit of $25, a planned reduction in work in 
process levels of 50%, and an average number of units in work in process per station of 100.  Assume 
that the total number of instances in which some work station goes out of control limits and produces 
defects is expected to be 500 annually and that in half those instances the out­of­control condition is 
not discovered immediately and enters 10% of the units produced.  The expected annual savings in 
cost of defective units would be:
A. $1,250
B. $31,250
C. $6,250
D. $125
E. none of the above

SUPPORTING CALCULATION:

$25 x (50% x 100 x 10%) x (1/2 x 500) = $31,250

D 16. The costs to be offset against the savings from lower work in process levels in a JIT system include all
of the following, except:
A. handling a larger number of small batches of work in process
B. the higher probability of shutdowns due to the smaller safety stock
C. the possibility that setup costs cannot be reduced enough to offset the larger number of setups
D. the possibility of customer dissatisfaction due to slower response time to orders
E. all of the above

A 17. Advantages that result from reducing raw materials inventory include all of the following except:
A. a decreased possibility of not being able to produce a unit when required
B. a need for less storage space
C. a reduced risk of obsolescence
D. a reduced risk of damaged materials
E. all of the above are advantages

C 18. Under a JIT approach to purchasing, the ideal number of vendors for each material is:
A. two
B. less than six
C. one
D. as many as can supply quality goods
E. none of the above

E 19. All of the following are obstacles to JIT purchasing, except:
A. the layout of the production process
B. the frequency of schedule changes
C. the attitudes of purchasing agents and suppliers
D. the distance from suppliers
E. all of the above are obstacles
Just­in­Time and Backflushing 94

B 20. JIT purchasing eliminates all of the following documents, except:
A. purchase requisitions
B. blanket purchase orders
C. receiving reports
D. materials requisitions
E. all of the above are eliminated

B 21. All of the following statements apply to a JIT work cell except that:
A. a cell is responsible for the entire production of a product or part
B. every worker in the cell specializes in a single task
C. a cell's workers may be evaluated and rewarded as a team
D. all workers in a cell are responsible for product quality
E. all of the above statements apply

A 22. All of the following are JIT performance measures, except:
A. capacity utilization
B. cycle time efficiency
C. inventory turnover
D. unscheduled maintenance downtime
E. number of defects

E 23. The cost accounting system that is noted for its lack of detailed tracking of work in process during the 
accounting period is:
A. process costing
B. job order costing
C. standard costing
D. actual costing
E. backflush costing

A 24. The cost accounting system that would be most apt to use a single inventory account entitled Raw and 
In Process (RIP) would be:
A. backflush costing
B. process costing
C. job order costing
D. historical costing
E. standard costing

D 25. To backflush materials cost from Raw and In Process (RIP) to Finished Goods, the calculation would 
be:
A. materials in ending RIP inventory plus materials received during the period minus materials in 
the beginning RIP inventory
B. materials in ending finished goods inventory plus materials cost transferred from RIP minus 
materials in beginning finished goods inventory
C. materials in beginning finished goods inventory plus materials cost transferred from RIP minus 
materials in ending finished goods inventory
D. materials in beginning RIP inventory plus materials received during the period minus materials 
in ending RIP inventory
E. none of the above
Just­in­Time and Backflushing 95

B 26. Cheeta Company has materials cost in the June 1 Raw and In Process of $10,000, materials received 
during June of $205,000 and materials cost in the June 30 Raw and In Process of $12,500.  The 
amount to be backflushed from Raw and In Process to Finished Goods at the end of June would be:
A. $215,000
B. $202,500
C. $207,500
D. $217,500
E. none of the above

D 27. In backflush costing, if the conversion cost in the Raw and In Process was $500 on July 1 and $1,000 
on July 31, the account to be credited at the end of July for the $500 increase would be:
A. Raw and In Process
B. Finished Goods
C. Raw Materials
D. Cost of Goods Sold
E. none of the above

A 28. In backflush costing, if the conversion cost in Raw and In Process was $1,000 on March 1 and $400 on
March 31, the account to be credited for the $600 decrease would be:
A. Raw and In Process
B. Finished Goods
C. Raw Materials
D. Cost of Goods Sold
E. none of the above

FACTORY OVERHEAD:

E 1. All of the following phrases are used as alternate terminology for "factory overhead" except:
A. manufacturing expense
B. indirect manufacturing cost
C. factory expense
D. factory burden
E. other expense

C 2. The component of per­unit costs that remains constant as the production level varies is:
A. general and administrative expenses
B. commercial expenses
C. variable factory overhead
D. fixed factory overhead
E. heat, light, and power

D 3. To express factory overhead as a percentage of direct materials dollars, estimated factory overhead is 
divided by estimated:
A. machine hours
B. normal capacity
C. units of materials used
D. materials cost
E. materials requisition usage

95
Just­in­Time and Backflushing 96

E 4. Estimated factory overhead is $600,000, and the hours usage of machinery is expected to be 150,000. 
Factory overhead is applied at the rate of $10 per direct labor hour.  The wage rate for direct labor is 
$6 per hour, and the total number of estimated direct labor hours for the period is:
A. 100,000
B. 150,000
C. 300,000
D. 600,000
E. 60,000

96
Factory Overhead:  Planned, Actual, and Applied 97

SUPPORTING CALCULATION:

B 5. An objection to the use of a factory overhead rate based on direct labor dollars is that:
A. these items are difficult to measure
B. a job is charged with more overhead when a highly paid operator works on the job than when a 
low­paid operator performs the work
C. overhead is allocated in relation to units produced by workers
D. overhead rates will be distributed inequitably when there are no wage differentials in the 
department 
E. costs of applying this method are excessive

D 6. A company expects to produce an average of 75,000 units per year, but last year production equaled 
60,000 units.  For the coming year, estimated production is 90,000 units.  Estimated overhead costs are
$900,000, and overhead is applied at the rate of $10 per unit.  The company bases its overhead rates 
on:
A. theoretical (engineering) capacity
B. a short­term planning approach
C. historical capacity costs
D. expected actual capacity
E. normal capacity

C 7. Direct costing differs from absorption costing in that:
A. direct materials and direct labor do not become a part of product cost under direct costing
B. the variable portion of overhead cost does not become a part of product cost under direct 
costing
C. the fixed portion of overhead cost does not become a part of product cost under direct costing
D. marketing and administrative expenses become a part of product cost under direct costing
E. direct costing does not differ from absorption costing

E 8. Application rates for factory overhead best reflect anticipated fluctuations in sales over a cycle of 
years when they are computed under the concept of:
A. practical capacity
B. expected actual capacity
C. theoretical capacity
D. maximum capacity
E. normal capacity

D 9. Underapplied factory overhead related to a significant decrease in production should be charged to:
A. Finished Goods Inventory
B. Cost of Goods Sold
C. Work in Process Inventory and Finished Goods Inventory
D. Work in Process Inventory, Finished Goods Inventory, and Cost of Goods Sold
E. Work in Process Inventory
Factory Overhead:  Planned, Actual, and Applied 98

B 10. Brownfield Company applies factory overhead on the basis of direct labor hours. Budget and actual 
data for direct labor and overhead for the year are as follows:

..................................................................................................... Budget   Actual  


Direct labor hours................................................................................... 600,000 550,000
Factory overhead costs........................................................................... $720,000 $640,000

The factory overhead for Brownfield for the year is:
A. underapplied by $40,000
B. overapplied by $20,000
C. overapplied by $40,000
D. underapplied by $20,000
E. neither underapplied nor overapplied

SUPPORTING CALCULATION:

D 11. A company manufactures plastic products for the home and restaurant market. The company also  does
contract work for other customers and utilizes a job order costing system. The flexible budget covering 
next year's expected range of activity is:

Direct labor hours..................................................... 50,000 80,000 110,000


Machine hours.......................................................... 40,000 64,000 88,000
Variable overhead costs............................................ $100,000 $160,000 $220,000
Fixed overhead costs................................................   150,000   150,000   150,000
Total overhead costs................................................. $250,000 $310,000 $370,000

A predetermined overhead rate based on direct labor hours at expected actual capacity is used to apply 
total overhead. Management has estimated that 100,000 direct labor hours will be used next year. The 
predetermined overhead rate per direct labor hour to be used to apply total overhead to individual jobs 
next year is:
A. $3.70
B. $3.88
C. $3.36
D. $3.50
E. none of the above

SUPPORTING CALCULATION:

$310,000 + $2(100,000 ­ 80,000)
Factory Overhead:  Planned, Actual, and Applied 99

C 12. At the end of the last fiscal year, Tiger Company had the following account balances:

Overapplied Overhead........................................................................................................ $    1,000
Cost of Goods Sold............................................................................................................. 980,000
Work in Process Inventory.................................................................................................. 38,000
Finished Goods Inventory................................................................................................... 82,000

The most common treatment of the Overapplied Overhead would be to:
A. carry it as a deferred credit on the balance sheet
B. report it as a miscellaneous operating revenue on the income statement
C. credit it to Cost of Goods Sold
D. prorate it between Work in Process Inventory and Finished Goods Inventory
E. prorate it among Work in Process Inventory, Finished Goods Inventory, and Cost of Goods Sold

A 13. Overapplied factory overhead would result if:
A. factory overhead costs incurred were less than costs charged to production
B. factory overhead costs incurred were unreasonably large in relation to units produced
C. factory overhead costs incurred were greater than costs charged to production
D. theoretical capacity were used in computing the overhead rate
E. the plant were operating at less than normal capacity 

D 14. Clyde Company found that the differences in product costs resulting from the application of 
predetermined overhead rates rather than actual overhead rates were very significant when actual 
production was substantially less than planned production. The most likely explanation is that:
A. costs of overhead were substantially less than anticipated 
B. overhead was composed chiefly of variable costs
C. several products were produced simultaneously 
D. fixed factory overhead was a significant cost
E. costs of overhead items were substantially higher than anticipated 

A 15. Avery Co. uses a predetermined factory overhead rate based on direct labor hours. For the month of 
October, Avery's budgeted overhead was $300,000 based on a budgeted volume of 100,000 direct labor 
hours. Actual overhead amounted to $325,000 with actual direct labor hours totaling 110,000. How 
much was the overapplied or underapplied overhead?
A. $5,000 overapplied
B. $5,000 underapplied
C. $30,000 overapplied
D. $30,000 underapplied
E. none of the above

SUPPORTING CALCULATION:
Factory Overhead:  Planned, Actual, and Applied 100

A 16. The absolute maximum capacity possible under the best conceivable operating conditions is a 
description of which type of activity level used in the computation of overhead rates?
A. theoretical
B. normal
C. practical
D. expected actual
E. currently attainable (expected)

B 17. All of the following are terms used to describe the phenomenon measured in the denominator of an 
overhead rate, except the:
A. base
B. overhead cost base
C. overhead rate base
D. overhead allocation base
E. all of the above are acceptable terms

B 18. The budget for a given factory overhead cost during a given period was $80,000.  The actual cost for the
period was $72,000.  Considering these facts, it can be said that the plant manager has done a better­
than­expected job in controlling the cost if:
A. the cost is a discretionary fixed cost and actual production equaled budgeted production
B. the cost is variable and actual production equaled budgeted production
C. the cost is variable and actual production was 90% of budgeted production
D. the cost is fixed and the actual production was less than budgeted production
E. the cost is variable and actual production was 80% of budgeted production

B 19. In highly automated manufacturing, all of the following may be appropriate bases for factory overhead 
application except:
A. machine hours
B. direct labor hours
C. number of setups
D. number of inspections
E. movement of materials

E 20. The transactions­base approach to overhead application gives particular consideration to:
A. the amount of direct labor cost
B. the number of machine hours
C. overhead costs that are not driven by volume of output 
D. special, high­volume production items
E. homogeneous production processes

D 21. When the amount of overapplied factory overhead is significant, the entry to close Overapplied Factory 
Overhead will most likely require:
A. a debit to Cost of Goods Sold
B. debits to Cost of Goods Sold, Finished Goods Inventory, and Work in Process Inventory
C. a credit to Cost of Goods Sold
D. credits to Cost of Goods Sold, Finished Goods Inventory, and Work in Process Inventory
E. none of the above
Factory Overhead:  Planned, Actual, and Applied 101

A 22. The type of activity level that results when theoretical capacity is reduced by allowances for 
unavoidable interruptions is:
A. practical capacity
B. expected actual capacity
C. normal capacity
D. excess capacity
E. none of the above

D 23. The condition that results either from greater productive capacity than the company could ever hope to
use or from an imbalance in equipment or machinery is termed:
A. theoretical capacity
B. practical capacity
C. idle capacity
D. excess capacity
E. none of the above

B 24. The method of product costing in which only variable overhead is included in the overhead rate is:
A. absorption costing
B. direct costing
C. conventional costing
D. full costing
E. none of the above

C 25. All of the following are names for the product costing method in which both fixed and variable costs 
are included in overhead rates, except:
A. absorption costing
B. conventional costing
C. direct costing
D. full costing
E. all of the above

DEPARTMENTALIZATION:

1. A department that would be classified as a producing department is:
A. Production Control
B. Utilities
C. Finishing
D. Medical
E. Shipping

B 2. A department that would be classified as a service department is:
A. Refining
B. Receiving
C. Mixing
D. Assembly
E. Finishing

A 3. In determining the right method for allocating equipment depreciation to departments, the best 
recommendation is to:
A. use the cost of equipment in the department as a basis for allocation

101
Factory Overhead:  Planned, Actual, and Applied 102

B. allocate on the basis of square footage used in a given department
C. charge the amounts to General Plant
D. use algebraic techniques
E. allocate on the basis of companywide rates

A 4. The most reasonable basis for allocating worker's compensation insurance is:
A. departmental payroll
B. building depreciation
C. kilowatt­hours
D. number of employees
E. materials used

E 5. A company is attempting to allocate the costs of electricity in various departments.  The variable 
portion of electricity expense is to be allocated using kilowatt­hours.  The information needed in order 
to allocate the fixed portion of the current period's electricity expense is:
A. rated horsepower of equipment
B. number of machines in each department
C. estimated materials consumption
D. number of employees
E. square footage in each department

102
Factory Overhead:  Departmentalization 103

E 6. The method for allocating service department costs that requires the least clerical work is:
A. use of square footage in each department
B. step method
C. allocation to other service departments only
D. simultaneous method
E. direct method

E 7. Rapid Falls Corp. has three producing departments, A, B, and C, with 50, 30, and 20 employees, 
respectively, in each department.  Factory payroll costs other than direct labor are accumulated in a 
Payroll Department account and are assigned to producing departments on the basis of number of 
employees.  The total payroll in each department was:  A, $300,000; B, $275,000; C, $325,000; and 
Payroll, $50,000.  Other costs accumulated in the Payroll Department amounted to $200,000.  The 
amount of Payroll Department costs chargeable to Department C is:
A. $125,000
B. $100,000
C. $40,000
D. $10,000
E. $50,000

SUPPORTING CALCULATION:

E 8. The following statement that best describes cost allocation is:
A. a company, as a general rule, should allocate indirect costs randomly or based on an "ability­to­
bear" criterion
B. a company can affect total income the most strongly by using the algebraic method of 
allocating indirect costs
C. a company can maximize or minimize total company income by selecting different bases on 
which to allocate indirect costs
D. a company should select an allocation base to raise or lower reported income on given products
E. a company's total income will remain unchanged no matter how indirect costs are allocated
Factory Overhead:  Departmentalization 104

D 9. Carmichael Manufacturing Company has two production departments (Fabrication and Assembly) and
three service departments (General Factory Administration, Factory Maintenance, and Factory 
Cafeteria). A summary of the year's costs and other data for each department prior to allocation of 
service department costs appears below.

General Factory )
Fabrication Assembly  Administration  )
Labor costs.............................................. $1,950,000 $ 2,050,000 $90,000 )
Material costs.......................................... $3,130,000 $ 950,000 )
Overhead................................................. $1,650,000 $ 1,850,000 $70,000 )
Direct labor hours.................................... 562,500 437,500 31,000 )
Number of employees............................. 280 200 12 )
Square footage occupied......................... 88,000 72,000 1,750 )

( Factory Factory
( Maintenance Cafeteria
( $82,100 $87,000
( $65,000 $91,000
( $56,100 $62,000
( 27,000 42,000
( 8 20
( 2,000 4,800

The costs of the General Factory Administration Department, Factory Maintenance Department, and 
Factory Cafeteria are allocated on the basis of direct labor hours, square footage occupied, and number
of employees, respectively.  There are no manufacturing overhead variances.
Assuming that Carmichael elects to distribute service department costs under the direct method of
cost allocation, the amount of Factory Maintenance Department costs that would be allocated to the 
Fabrication Department is (round all final calculations to the nearest dollar):
A. $106,091
B. $91,440
C. $0
D. $111,760
E. none of the above

SUPPORTING CALCULATION:
Factory Overhead:  Departmentalization 105

A 10. Carmichael Manufacturing Company has two production departments (Fabrication and Assembly) and
three service departments (General Factory Administration, Factory Maintenance, and Factory 
Cafeteria).  A summary of the year's costs and other data for each department prior to allocation of 
service department costs appears below.

General Factory )
Fabrication Assembly  Administration  )
Labor costs.............................................. $1,950,000 $2,050,000 $90,000 )
Material costs.......................................... $3,130,000 950,000 )
Overhead................................................. $1,650,000 $1,850,000 $70,000 )
Direct labor hours.................................... 562,500 437,500 31,000 )
Number of employees............................. 280 200 12 )
Square footage occupied......................... 88,000 72,000 1,750 )

( Factory Factory
( Maintenance Cafeteria
( $82,100 $87,000
( $65,000 $91,000
( $56,100 $62,000
( 27,000 42,000
( 8 20
( 2,000 4,800

The costs of the General Factory Administration Department, Factory Maintenance Department, and 
Factory Cafeteria are allocated on the basis of direct labor hours, square footage occupied, and number
of employees, respectively.
The amount of General Factory Administration Department costs that would be allocated to the 
Assembly Department under the direct method is (round all final calculations to the nearest dollar):
A. $70,000
B. $90,000
C. $0
D. $63,636
E. none of the above

SUPPORTING CALCULATION:
Factory Overhead:  Departmentalization 106

B 11. Carmichael Manufacturing Company has two production departments (Fabrication and Assembly) and
three service departments (General Factory Administration, Factory Maintenance, and Factory 
Cafeteria).  A summary of the year's costs and other data for each department prior to allocation of 
service department costs appears below.

General Factory )
Fabrication Assembly  Administration  )
Labor costs.............................................. $1,950,000 $2,050,000 $90,000 )
Material costs.......................................... $3,130,000 $950,000 )
Overhead................................................. $1,650,000 $1,850,000 $70,000 )
Direct labor hours.................................... 562,500 437,500 31,000 )
Number of employees............................. 280 200 12 )
Square footage occupied......................... 88,000 72,000 1,750 )

( Factory Factory
( Maintenance Cafeteria
( $82,100 $87,000
( $65,000 $91,000
( $56,100 $62,000
( 27,000 42,000
( 8 20
( 2,000 4,800

The costs of the General Factory Administration Department, Factory Maintenance Department, and 
Factory Cafeteria are allocated on the basis of direct labor hours, square footage occupied, and number
of employees, respectively.
Assuming that Carmichael elects to distribute service department costs to other service 
departments using the step method of cost allocation and that the order of distribution is based on the 
dollar amount of costs originating in the service departments, how much of the total Factory Cafeteria 
cost would be allocated to the Factory Maintenance Department?  (Round all final calculations to the 
nearest dollar.)
A. $96,000
B. $3,840
C. $6,124
D. $0
E. none of the above

SUPPORTING CALCULATION:
Factory Overhead:  Departmentalization 107

B 12. Carmichael Manufacturing Company has two production departments (Fabrication and Assembly) and
three service departments (General Factory Administration, Factory Maintenance, and Factory 
Cafeteria).  A summary of the year's costs and other data for each department prior to allocation of 
service department costs appears below.

General Factory )
Fabrication Assembly  Administration  )
Labor costs.............................................. $1,950,000 $2,050,000 $90,000 )
Material costs.......................................... $3,130,000 $950,000 )
Overhead................................................. $1,650,000 $1,850,000 $70,000 )
Direct labor hours.................................... 562,500 437,500 31,000 )
Number of employees............................. 280 200 12 )
Square footage occupied......................... 88,000 72,000 1,750 )

( Factory Factory
( Maintenance Cafeteria
( $82,100 $87,000
( $65,000 $91,000
( $56,100 $62,000
( 27,000 42,000
( 8 20
( 2,000 4,800

The costs of the General Factory Administration Department, Factory Maintenance Department, and 
Factory Cafeteria are allocated on the basis of direct labor hours, square footage occupied, and number
of employees, respectively.
How much of the Factory Maintenance Department costs would be allocated to the Factory 
Cafeteria under the step method, assuming that the order of distribution is based on the dollar amount 
of costs originating in the service departments? (Round all final calculations to the nearest dollar.)
A. $148,910
B. $0
C. $5,787
D. $5,856
E. none of the above

SUPPORTING CALCULATION:

Factory Cafeteria costs........................................................................................................ $240,000
Factory Maintenance costs.................................................................................................. $203,300
  Factory Cafeteria already closed out.
Factory Overhead:  Departmentalization 108

A 13. Acie Company has two service departments and three production departments, each producing a 
separate product.  For a number of years, Acie has allocated the costs of the service departments to the 
production departments on the basis of the annual sales dollars.  In a recent audit report, the internal 
auditor stated that the distribution of service department costs on the basis of annual sales dollars 
would lead to serious inequities.  It was recommended that maintenance and engineering service hours
be used as a better service cost allocation basis.  For illustration purposes, the following information 
was appended to the audit report:

Service Departments )
Maintenance Engineering )
Maintenance hours used........................................ 400 )
Engineering hours used......................................... 400 )
Department direct costs......................................... $12,000 $54,000 )

( Production Departments
( Department A Department B Department C
( 800 200 200
( 800 400 400
( $80,000 $90,000 $50,000

Using the simultaneous method, what would be the total Engineering Department cost after allocation 
of interservice department costs, but before allocation to the Maintenance and Production 
Departments?
A. $60,000
B. $57,000
C. $12,000
D. $54,000
E. none of the above

SUPPORTING CALCULATION:

Maintenance = $12,000 + .2E
Engineering = $54,000 + .25M
E = $54,000 + .25($12,000 + .2E)
E = $54,000 + $3,000 + .05E
.95E = $57,000
E = $60,000
Factory Overhead:  Departmentalization 109

D 14. Acie Company has two service departments and three production departments, each producing a 
separate product.  For a number of years, Acie has allocated the costs of the service departments to the 
production departments on the basis of the annual sales dollars.  In a recent audit report, the internal 
auditor stated that the distribution of service department costs on the basis of annual sales dollars 
would lead to serious inequities.  It was recommended that maintenance and engineering service hours
be used as a better service cost allocation basis.  For illustration purposes, the following information 
was appended to the audit report:

Service Departments )
Maintenance Engineering )
Maintenance hours used........................................ 400 )
Engineering hours used......................................... 400 )
Department direct costs......................................... $12,000 $54,000 )

( Production Departments
( Department A Department B Department C
( 800 200 200
( 800 400 400
( $80,000 $90,000 $50,000

Using the simultaneous method, what would be the total Maintenance Department cost after allocation
of interservice department costs, but before allocation to the Engineering and Production 
Departments?
A. $72,000
B. $12,000
C. $60,000
D. $24,000
E. none of the above

SUPPORTING CALCULATION:

Maintenance = $12,000 + .2E
Engineering = $54,000 + .25M
M = $12,000 + .2E
M = $12,000 + .2($54,000 + .25M)
M = $12,000 + $10,800 + .05M
.95M = $22,800
M = $24,000
Factory Overhead:  Departmentalization 110

C 15. Acie Company has two service departments and three production departments, each producing a 
separate product.  For a number of years, Acie has allocated the costs of the service departments to the 
production departments on the basis of the annual sales dollars.  In a recent audit report, the internal 
auditor stated that the distribution of service department costs on the basis of annual sales dollars 
would lead to serious inequities.  It was recommended that maintenance and engineering service hours
be used as a better service cost allocation basis.  For illustration purposes, the following information 
was appended to the audit report:

Service Departments )
Maintenance Engineering )
Maintenance hours used........................................ 400 )
Engineering hours used......................................... 400 )
Department direct costs......................................... $12,000 $54,000 )

( Production Departments
( Department A Department B Department C
( 800 200 200
( 800 400 400
( $80,000 $90,000 $50,000

Using the step method of cost allocation, what amount of maintenance cost would be allocated to 
Department A, assuming that the service departments are distributed in the order of total dollars of 
direct departmental costs?
A. $0
B. $25,500
C. $15,200
D. $3,187.50
E. none of the above

SUPPORTING CALCULATION:

Maintenance = $12,000 + .2($54,000) = $22,800
Department A = 800/1,200 x $22,800 = $15,200
Factory Overhead:  Departmentalization 111

B 16. Acie Company has two service departments and three production departments, each producing a 
separate product.  For a number of years, Acie has allocated the costs of the service departments to the 
production departments on the basis of the annual sales dollars.  In a recent audit report, the internal 
auditor stated that the distribution of service department costs on the basis of annual sales dollars 
would lead to serious inequities.  It was recommended that maintenance and engineering service hours
be used as a better service cost allocation basis.  For illustration purposes, the following information 
was appended to the audit report:

Service Departments )
Maintenance Engineering )
Maintenance hours used........................................ 400 )
Engineering hours used......................................... 400 )
Department direct costs......................................... $12,000 $54,000 )

( Production Departments
( Department A Department B Department C
( 800 200 200
( 800 400 400
( $80,000 $90,000 $50,000

Using the step method of cost allocation, what amount of engineering cost would be allocated directly 
to Department A, assuming that the service departments are distributed in the order of total dollars of 
direct departmental costs?
A. $11,400
B. $21,600
C. $10,800
D. $22,800
E. none of the above

SUPPORTING CALCULATION:

800/2,000 x $54,000 = $21,600

E 17. A factor to be considered in deciding the kinds of departments required for establishing accurate 
departmental overhead rates with which to control costs is:
A. location of operations, processes, and machinery
B. responsibilities for production and costs
C. number of departments or cost centers
D. similarity of operations, procedures, and machinery in each department
E. all of the above

E 18. Services available for the benefit of producing departments and other service departments can be 
organized by:
A. establishing a separate service department for each function
B. combining several functions into one department
C. placing service costs in a department called "general factory cost pool"
D. none of the above
E. all of the above
Factory Overhead:  Departmentalization 112

B 19. Entities that have practiced departmentalization for many years, by grouping their activities into 
categories such as occupancy, sales promotion, purchasing, and delivery are:
A. hospitals
B. retail stores
C. banks
D. insurance companies
E. colleges

A 20. An automotive company has three divisions.  One division manufactures new replacement parts for 
automobiles; another rebuilds engines; and the third does repair and overhaul work on a line of trucks. 
All three divisions use the services of a central payroll department.  The best method of allocating the 
cost of the payroll department to the various operating divisions is:
A. total labor hours incurred in the divisions
B. value of production in the divisions
C. direct materials costs incurred in the divisions
D. machine hours used in the divisions
E. none of the above

B 21. The Janitorial Department provides cleaning services to all departments of a large store.  Management 
wishes to allocate the janitorial costs to the various departments that benefit from the service.  The 
most reasonable allocation base for janitorial costs would be:
A. sales of each department
B. square footage of each department
C. number of employees in each department
D. total direct costs of each department before any allocations
E. none of the above

C 22. A hospital has a $100,000 expected utility bill this year.  The Janitorial, Accounting, and Orderlies 
Departments are service functions to the Operating, Hospital Rooms, and Laboratories Departments.  
Floor space assigned to each department is:

Department...................................................................................................... Square Footage
Janitorial.......................................................................................................... 1,000
Accounting....................................................................................................... 2,000
Orderlies.......................................................................................................... 7,000
Operating......................................................................................................... 4,000
Hospital Rooms............................................................................................... 30,000
Laboratories.....................................................................................................   6,000
.............................................................................................................. 50,000

How much of the $100,000 will eventually become the Hospital Rooms Department total costs, 
assuming use of the direct method of allocation based on square footage?
A. $60,000
B. $72,000
C. $75,000
D. $80,000
E. none of the above
Factory Overhead:  Departmentalization 113

SUPPORTING CALCULATION:

C 23. Serpent Corp. distributes service department overhead costs directly to producing departments without
allocation to the other service department.  Information for the month of June is as follows:

Service Departments
Maintenance Utilities
Overhead costs incurred........................................................... $20,000 $10,000
Service provided to department:
Maintenance...................................................................... ­­  10%
Utilities.............................................................................. 20% ­­ 
ProducingCA..................................................................... 40% 30%
ProducingCB.....................................................................  40%  60%
Totals........................................................................................ 100% 100%

The amount of Maintenance Department costs distributed to ProducingCA Department for June was:
A. $8,000
B. $8,800
C. $10,000
D. $11,000
E. none of the above

SUPPORTING CALCULATION:

D 24. Multiple overhead rates are most commonly used when:
A. production consists of long runs of a single product
B. the company has more than one production department
C. manufacturing operations are labor intensive
D. production consists of a diverse product line
E. none of the above

B 25. An example of a nonvolume­related overhead base would be:
A. direct materials cost
B. number of setups
C. machine hours
D. direct labor dollars
E. none of the above
Factory Overhead:  Departmentalization 114

C 26. An example of a department that would be a prime candidate for multiple overhead rates would be one
whose overhead was primarily:
A. labor driven
B. machine related
C. caused by setups and production design changes
D. materials related
E. none of the above
Factory Overhead:  Departmentalization 115

ABC:

A 1. A base used to allocate the cost of a resource to the different activities using that resource is a(n):
A. resource driver
B. activity driver
C. final cost object
D. driver
E. none of the above

B 2. A base used to allocate the cost of products, customers, or other final cost objects is a(n):
A. resource driver
B. activity driver
C. final cost object
D. driver
E. none of the above

B 3. Examples of activities at the batch level of costs include:
A. cutting, painting, and packaging
B. scheduling, setting up, and moving
C. designing, changing, and advertising
D. heating, lighting, and security
E. none of the above

C 4. Examples of activities at the product level of costs include:
A. cutting, painting, and packaging
B. scheduling, setting up, and moving
C. designing, changing, and advertising
D. heating, lighting, and security
E. none of the above

D 5. Examples of activities at the plant level of costs include:
A. cutting, painting, and packaging
B. scheduling, setting up, and moving
C. designing, changing, and advertising
D. heating, lighting, and security
E. none of the above

115
116 Chapter 14

A 6. Examples of activities at the unit level of costs include:
A. cutting, painting, and packaging
B. scheduling, setting up, and moving
C. designing, changing, and advertising
D. heating, lighting, and security
E. none of the above

A 7. Examples of unit­level costs are:
A. portions of electricity and indirect materials
B. salaries of schedulers and setup personnel
C. salaries of designers and programmers
D. depreciation and insurance on buildings
E. none of the above

C 8. Examples of product­level costs are:
A. portions of electricity and indirect materials
B. salaries of schedulers and setup personnel
C. salaries of designers and programmers
D. depreciation and insurance on buildings
E. none of the above

D 9. Examples of plant­level costs are:
A. portions of electricity and indirect materials
B. salaries of schedulers and setup personnel
C. salaries of designers and programmers
D. depreciation and insurance on buildings
E. none of the above

B 10. Examples of batch­level costs are:
A. portions of electricity and indirect materials
B. salaries of schedulers and setup personnel
C. salaries of designers and programmers
D. depreciation and insurance on buildings
E. none of the above

A 11. Examples of unit­level activity drivers include:
A. units of output and direct labor hours
B. number of batches and material moves
C. number of products and design changes
D. square footage occupied
E. all of the above

B 12. Examples of batch­level activity drivers include:
A. units of output and direct labor hours
B. number of batches and material moves
C. number of products and design changes
D. square footage occupied
E. all of the above
Activity Accounting:  Activity­Based Costing and Activity­Based Management 117

C 13. Examples of product­level activity drivers include:
A. units of output and direct labor hours
B. number of batches and material moves
C. number of products and design changes
D. square footage occupied
E. all of the above

D 14. Examples of plant­level activity drivers include:
A. units of output and direct labor hours
B. number of batches and material moves
C. number of products and design changes
D. square footage occupied
E. all of the above

A 15. Unit­level costs are costs that:
A. inevitably increase whenever a unit is produced
B. are caused by the number of batches produced and sold
C. are incurred to support the number of different products produced
D. are incurred to sustain capacity at a production site
E. none of the above

D 16. Plant­level costs are costs that:
A. inevitably increase whenever a unit is produced
B. are caused by the number of batches produced and sold
C. are incurred to support the number of different products produced
D. are incurred to sustain capacity at a production site
E. none of the above

E 17. Unit­level drivers are:
A. inversely proportional to the volume of output
B. measures of activities that vary with the number of batches produced and sold
C. measures of activity that vary with the number of different products produced and sold
D. for assigning plant­level costs
E. none of the above

A 18. Traditional costing systems are characterized by their use of which of the following measures as bases 
for allocating overhead to output:
A. unit­level drivers
B. batch­level drivers
C. product­level drivers
D. plant­level drivers
E. none of the above

E 19. ABC systems are characterized by their use of which of the following measures as bases for allocating 
overhead to output:
A. unit­level drivers
B. batch­level drivers
C. product­level drivers
D. plant­level drivers
E. all of the above
Activity Accounting:  Activity­Based Costing and Activity­Based Management 118

A 20. All of the following are distinctions that usually exist between traditional and ABC costing systems, 
except that:
A. the number of overhead cost pools tends to be lower in ABC systems
B. the number of allocation bases tend to be higher in ABC systems
C. costs within an ABC cost pool tend to be more homogeneous than the costs within a traditional 
system's cost pool
D. all ABC systems are two­stage costing systems, while traditional systems may be one­ or two­
stage
E. all of the above are distinctions

D 21. All of the following are distinctions that usually exist between traditional and ABC costing systems, 
except that:
A. the number of overhead cost pools tends to be higher in ABC systems
B. the number of allocation bases tend to be higher in ABC systems
C. costs within an ABC cost pool tend to be more homogeneous than the costs within a traditional 
system's cost pool
D. all ABC systems are one­stage costing systems, while traditional systems may be one­ or two­
stage
E. all of the above are distinctions

B 22. Compared to an ABC system, a traditional costing system reports:
A. a lower unit cost for high­volume products and a higher unit cost for low­volume products
B. a higher unit cost for high­volume products and a lower unit cost for low­volume products
C. the same unit costs for high­ and low­volume products as does an ABC system
D. either higher or lower unit cost for high­volume products than an ABC system depending upon 
the level of fixed costs
E. none of the above

C 23. Activity­based management (ABM) is:
A. a costing system in which multiple overhead cost pools are allocated using bases that include 
one or more nonvolume related factors
B. a base used to allocate the cost of a resource to the different activities using it
C. the use of information obtained from ABC to make improvements in the firm
D. a base used to allocate the cost of an activity to products and customers
E. none of the above

A 24. All of the following are ways that activities can be managed to achieve improvements in a process, 
except:
A. activity induction
B. activity elimination
C. activity selection
D. activity sharing
E. all of the above are ways in which activities may be managed

C 25. All of the following are examples of non­value­added activities except:
A. ordering
B. receiving
C. assembling
D. inspections
E. setting up
Activity Accounting:  Activity­Based Costing and Activity­Based Management 119

STANDARD:

D 1. The type of standard that is intended to represent challenging yet attainable results is:
A. theoretical standard
B. flexible budget standard
C. controllable cost standard
D. normal standard
E. expected actual standard

A 2. Standard costs are used for all of the following except:
A. income determination
B. controlling costs
C. measuring efficiencies
D. forming a basis for price setting
E. establishing budgets

C 3. Of the following variances, the one that is most useful in assessing the performance of the Purchasing 
Department is the:
A. idle capacity variance
B. overhead price variance
C. materials purchase price variance
D. labor rate variance
E. materials price usage variance

B 4. The labor efficiency variance is computed as:
A. the difference between standard and actual rates, multiplied by standard hours
B. the difference between standard and actual hours, multiplied by standard rate
C. the difference between standard and actual rates, multiplied by actual hours
D. the difference between standard and actual hours, multiplied by the difference between standard
and actual rates
E. a percentage of the labor time variance

119
120 Chapter 18

B 5. The method used to assure fairness in the rates paid for each operation performed by an employee is:
A. job costing
B. job rating
C. union contracting
D. the agreed­upon wages at the time of employment
E. labor rate variance analysis

D 6. Materials and labor cost standards are generally based on:
A. expected actual conditions, anticipated prices, and desired efficiency levels
B. theoretical conditions, present price levels, and desired efficiency levels
C. capacity conditions, anticipated prices, and desired efficiency levels
D. normal conditions, present price levels, and desired efficiency levels
E. theoretical conditions, anticipated prices, and theoretically attainable efficiency levels

D 7. The most effective standards are set following a careful study of products and operating conditions by 
the:
A. Accounting Department, central management, and the Industrial Engineering Department
B. central management and the employees whose performance is being evaluated
C. Accounting Department and engineering staff
D. Industrial Engineering Department and the employees whose performance is being evaluated
E. central management and the Industrial Engineering Department

E 8. In analyzing factory overhead variances, the volume variance is the difference between the:
A. actual amount spent for overhead items during the period and the amount applied during the 
period
B. variable efficiency variance and fixed efficiency variance
C. amount shown in the flexible budget and the amount shown in the master budget
D. master budget application rate and the flexible budget application rate, multiplied by actual 
hours worked
E. budget allowance based on standard hours allowed for actual production for the period and the 
amount of applied factory overhead during the period

D 9. The variance resulting from obtaining an output different from the one expected on the basis of input 
is the:
A. mix variance
B. output variance
C. usage variance
D. yield variance
E. efficiency variance

A 10. In its reports to management, a company disclosed the presence of a fixed efficiency variance.  The 
procedure used to analyze variances was the:
A. four­variance method
B. mix and yield variances method
C. two­variance method
D. alternative three­variance method
E. three­variance method
Standard Costing:  Setting Standards and Analyzing Variances 121

D 11. A purpose of standard costing is to:
A. allocate cost with more accuracy
B. eliminate the need for subjective decisions by management
C. determine the "break­even" production level
D. control costs
E. all of the above

A 12. Which one of the following is true concerning standard costs?
A. If properly used, standards can help motivate employees.
B. Unfavorable variances, material in amount, should be investigated, but large favorable 
variances need not be investigated.
C. Standard costs are difficult to use with a process costing system.
D. Standard costs are estimates of costs attainable only under the most ideal conditions, but rarely 
practicable.
E. All of the above

A 13. When computing variances from standard costs, the difference between actual and standard price 
multiplied by actual quantity yields a:
A. price variance
B. volume variance
C. mix variance
D. yield variance
E. combined price­quantity variance

E 14. A company controls its production costs by comparing its actual monthly production costs with the 
expected levels.  Any significant deviations from expected levels are investigated and evaluated as a 
basis for corrective actions.  The quantitative technique that is most probably being used is:
A. time­series or trend regression analysis
B. correlation analysis
C. differential calculus
D. risk analysis
E. standard cost variance analysis

C 15. What type of direct material variances for price and usage will arise if the actual number of pounds of 
materials used was less than standard pounds allowed but actual cost exceeds standard cost?

Usage Price
A. unfavorable favorable
B. favorable favorable
C. favorable unfavorable
D. unfavorable unfavorable
E. none none

B 16. If a company follows a practice of isolating variances at the earliest time, the appropriate time to 
isolate and recognize a direct materials price variance would be when:
A. the purchase order is originated
B. materials are purchased
C. materials are issued
D. the materials requisition is prepared
E. materials are used in production
Standard Costing:  Setting Standards and Analyzing Variances 122

A 17. Which of the following would least likely cause an unfavorable materials quantity (usage) variance?
A. labor that possesses skills equal to those required by the standards
B. scheduling of substantial overtime
C. a mix of direct materials that does not conform to plan
D. materials that do not meet specifications
E. machinery that has not been maintained properly

D 18. Information about Sargent Company's direct material costs is as follows:

Standard unit price $3.60
Actual quantity purchased 1,600
Standard quantity allowed for actual production 1,450
Materials purchase price varianceCunfavorable $240

What was the actual purchase price per unit, rounded to the nearest penny?
A. $3.06
B. $3.11
C. $3.45
D. $3.75
E. $3.60

SUPPORTING CALCULATION:

$240 = 1,600 (x ­ $3.60)
1,600 x = $240 + $5,760
x = $3.75

C 19. Using the following symbols, which formula represents the calculation of the labor rate variance?

AH = Actual hours
SH = Standard hours allowed for actual production
AR = Actual rate
SR = Standard rate

A. SR(AH ­ SH)
B. AR(AH ­ SH)
C. AH(AR ­ SR)
D. SH(AR ­ SR)
E. SH(SR ­ AR)

D 20. When a change in the manufacturing process reduces the number of direct labor hours and standards 
are unchanged, the resulting variance will be:
A. an unfavorable labor usage variance
B. an unfavorable labor rate variance
C. a favorable labor rate variance
D. a favorable labor usage variance
E. both (C) and (D) above
Standard Costing:  Setting Standards and Analyzing Variances 123

B 21. The most probable reason a company would experience a favorable labor rate variance and an 
unfavorable labor efficiency variance is that:
A. the mix of workers assigned to the particular job was heavily weighted toward the use of higher
paid, experienced individuals
B. the mix of workers assigned to the particular job was heavily weighted toward the use of new, 
relatively low­paid, unskilled workers
C. because of the production schedule, workers from other production areas were assigned to assist
in this particular process
D. defective materials caused more labor to be used in order to produce a standard unit
E. the actual price paid for materials that went into production was less than the standard price that
was expected to be paid

C 22. Information on Orman Company's direct labor costs is as follows:

Standard direct labor rate................................................................................................. $3.75
Actual direct labor rate..................................................................................................... $3.50
Standard direct labor hours.............................................................................................. 10,000
Direct labor usage (efficiency) varianceCunfavorable.................................................... $  4,200

What were the actual hours worked, rounded to the nearest hour?
A. 11,914
B. 10,714
C. 11,120
D. 11,200
E. none of the above

SUPPORTING CALCULATION:

$4,200 = $3.75 (x ­ 10,000)
$3.75 x = $4,200 + $37,500
x = 11,120

D 23. Each unit of Product 8in1 requires two direct labor hours. Employee benefit costs are treated as direct 
labor costs. Data on direct labor are as follows:

Number of direct employees............................................................................................ 25
Weekly productive hours per employee........................................................................... 30
Estimated weekly wages per employee............................................................................ $240
Employee benefits (related to weekly wages).................................................................. 25%

The standard direct labor cost per unit of Product 8in1 is:
A. $8.00
B. $10.00
C. $12.00
D. $20.00
E. none of the above

SUPPORTING CALCULATION:
Standard Costing:  Setting Standards and Analyzing Variances 124

B 24. J. R. Richard Company employs a standard absorption system for product costing.  The standard cost 
of its product is as follows:

Direct materials................................................................................................................ $14.50
Direct labor (2 direct labor hours x $8)............................................................................ 16.00
Manufacturing overhead (2 direct labor hours x $11)......................................................  22.00
Total standard cost........................................................................................................... $52.50

The manufacturing overhead rate is based upon a normal activity level of 600,000 direct labor hours.  
Richard planned to produce 25,000 units each month during the year.  The budgeted annual 
manufacturing overhead is:

Variable............................................................................................................................ $3,600,000
Fixed.................................................................................................................................  3,000,000
.......................................................................................................................................... $6,600,000

During November, Richard produced 26,000 units.  Richard used 53,500 direct labor hours in 
November at a cost of $433,350.  Actual manufacturing overhead for the month was $250,000 fixed 
and $325,000 variable.

The manufacturing overhead controllable variance for November is:
A. $9,000 unfavorable
B. $13,000 unfavorable
C. $9,000 favorable
D. $4,000 favorable
E. none of the above

SUPPORTING CALCULATION:

Actual factory overhead................................................... $ 575,000
Budget allowance:
Variable factory overhead (52,000 x $6).................. $312,000
Budgeted fixed overhead..........................................   250,000         562,000
Controllable variance.......................................................  $       
   13,000 unfavorable
Standard Costing:  Setting Standards and Analyzing Variances 125

B 25. J. R. Richard Company employs a standard absorption system for product costing.  The standard cost 
of its product is as follows:

Direct materials.............................................................................................................. $14.50
Direct labor (2 direct labor hours x $8)......................................................................... 16.00
Manufacturing overhead (2 direct labor hours x $11)...................................................    22.00
Total standard cost......................................................................................................... $52.50

The manufacturing overhead rate is based upon a normal activity level of 600,000 direct labor hours.  
Richard planned to produce 25,000 units each month during the year.  The budgeted annual 
manufacturing overhead is:

Variable.......................................................................................................................... $3,600,000
Fixed ............................................................................................................................   3,000,000
$6,600,000

During November, Richard produced 26,000 units.  Richard used 53,500 direct labor hours in 
November at a cost of $433,350.  Actual manufacturing overhead for the month was $250,000 fixed 
and $325,000 variable.

The manufacturing overhead volume variance for November is:
A. $12,000 unfavorable
B. $10,000 unfavorable
C. $3,000 unfavorable
D. $9,000 unfavorable
E. $1,000 favorable

SUPPORTING CALCULATION:

Budget allowance based on standard hours allowed 
[(52,000 x $6) + $250,000].............................................................. $ 562,000 
Factory overhead applied at standard.......................................................            572,000 
Volume variance.......................................................................................  $      
   (10,000) favorable

C 26. The following information relates to Department 1 of Ruiz Company for the fourth quarter.  The total 
overhead variance is divided into three variances: spending, variable efficiency, and volume.

Actual total overhead (fixed plus variable)....................................... $178,500
Budget formula.................................................................................. $110,000 + $.50 per hour
Total overhead application rate......................................................... $1.50 per hour
Actual hours worked.......................................................................... 121,000

What was the spending variance in this department during the quarter?
A. $8,000 favorable
B. $4,500 favorable
C. $8,000 unfavorable
D. $4,500 unfavorable
E. none of the above
Standard Costing:  Setting Standards and Analyzing Variances 126

SUPPORTING CALCULATION:

Actual factory overhead.................................................... $ 178,500
Budget allowance:
Variable for actual hours 
      (121,000 x $.50)................................................... $  60,500
Fixed..........................................................................   110,000    170,500
Spending variance.............................................................  $         
   8,000 unfavorable

A 27. The following information relates to Department 1 of Ruiz Company for the fourth quarter.  The total 
overhead variance is divided into three variances: spending, variable efficiency, and volume.

Actual total overhead (fixed plus variable)....................................... $178,500
Budget formula.................................................................................. $110,000 + $.50 per hour
Total overhead application rate......................................................... $1.50 per hour
Actual hours worked.......................................................................... 121,000
Standard hours allowed for production............................................. 130,000

What was the variable efficiency variance in this department during the quarter?
A. $4,500 favorable
B. $8,000 favorable
C. $4,500 unfavorable
D. $8,000 unfavorable
E. none of the above

SUPPORTING CALCULATION:

Budget allowance for actual hours 
[(121,000 x $.50) + $110,000]................................... $ 170,500 
Budget allowance for standard hours:
Variable (130,000 x $.50).......................................... $  65,000
Fixed..........................................................................   110,000         175,000 
Variable efficiency variance..............................................  $      
   (4,500) favorable

E 28. Under the two­variance method for analyzing factory overhead, the controllable (budget) variance is 
the difference between the:
A. actual fixed factory overhead and the budgeted fixed overhead
B. budget allowance based on standard hours allowed and the factory overhead applied to 
production
C. budget allowance based on standard hours allowed and the budget allowance based on actual 
hours worked
D. actual factory overhead and the factory overhead applied to production
E. actual factory overhead and the budget allowance based on standard hours allowed

A 29. Materials usage variances are normally chargeable to the:
A. Production Department
B. Purchasing Department
C. Finished Goods Department
D. Materials Storage Department
E. Factory Storeroom Department
Standard Costing:  Setting Standards and Analyzing Variances 127

C 30. Todco planned to produce 3,000 units of its single product, Teragram, during November.  The 
standard specifications for one unit of Teragram include six pounds of material at $.30 per pound.  
Actual production in November was 3,100 units of Teragram.  The accountant computed a favorable 
materials purchase price variance of $380 and an unfavorable materials quantity variance of $120.  
Based on these variances, one could conclude that:
A. more materials were purchased than were used
B. more materials were used than were purchased
C. the actual cost of materials was less than the standard cost
D. the actual usage of materials was less than the standard allowed
E. actual cost and usage of materials were both less than standard

D 31. Information on Duke Co.'s direct material costs for May is as follows:

Actual quantity of direct materials purchased and used................................................ 30,000 lbs.


Actual cost of direct materials....................................................................................... $84,000
Unfavorable direct materials usage variance................................................................. 3,000
Standard quantity of direct materials allowed for May production............................... 29,000 lbs.

For the month of May, Duke's direct materials price variance was:
A. $2,800 favorable
B. $2,800 unfavorable
C. $6,000 unfavorable
D. $6,000 favorable
E. none of the above

SUPPORTING CALCULATION:

$3,000 = x (30,000 ­ 29,000)
1,000 x = $3,000
x = $3
y = $2.80 ­ $3.00(30,000)
y = ($6,000) favorable

A 32. A company uses a standard cost system to account for its only product.  The materials standard per 
unit was 4 lbs. at $5.10 per lb.  Operating data for April were as follows:

Material used.................................................................................................................. 7,800 lbs.


Cost of material used..................................................................................................... $40,950
Number of finished units produced............................................................................... 2,000

The material usage variance for April was:
A. $1,020 favorable
B. $1,050 favorable
C. $1,170 unfavorable
D. $1,200 unfavorable
E. none of the above
Standard Costing:  Setting Standards and Analyzing Variances 128

SUPPORTING CALCULATION:

x = $5.10 [7,800 ­ (2,000 x 4)]
x = ($1,020) favorable

D 33. During the last three months, a manufacturer incurred an unfavorable labor efficiency variance. The 
least likely cause of this variance is:
A. substantial materials were purchased at a discount at a previously unused supplier's liquidation
B. for one week, only half of the workforce, those with the highest seniority, were called in to 
work
C. a second production line with all new personnel was started 
D. the cost­of­living adjustment for the three­month period was $.10 more per hour than expected
E. none of the above

D 34. The direct labor standards for producing a unit of a product are two hours at $10 per hour.  Budgeted 
production was 1,000 units.  Actual production was 900 units, and direct labor cost was $19,000 for 
2,000 direct labor hours.  The direct labor efficiency variance was:
A. $1,000 favorable
B. $1,000 unfavorable
C. $2,000 favorable
D. $2,000 unfavorable
E. none of the above

SUPPORTING CALCULATION:

x = $10 [2,000 ­ (900 x 2)]
x = $2,000 unfavorable

C 35. Under the two­variance method for analyzing factory overhead, the factory overhead applied to 
production is used in the computation of the:

Controllable Volume
(Budget) Variance Variance
A. yes no
B. yes yes
C. no yes
D. no no

D 36. Under the three­variance method for analyzing factory overhead, which of the following is used in 
computation of the spending variance?

Actual Factory Budget Allowance
   Overhead    Based on Actual Hours
A. no yes
B. no no
C. yes no
D. yes yes
Standard Costing:  Setting Standards and Analyzing Variances 129

D 37. Compute the variable efficiency variance, using the following data:

Standard labor hours per good unit produced....................................................................... 2
Good units produced............................................................................................................. 1,000
Actual labor hours used........................................................................................................ 2,100
Standard variable overhead per standard labor hour............................................................. $3
Actual variable overhead...................................................................................................... $ 6,500

A. $200 favorable
B. $200 unfavorable
C. $300 favorable
D. $300 unfavorable
E. none of the above

SUPPORTING CALCULATION:

Variable budget allowance for actual hours (2,100 x $3)................................ $ 6,300
Variable budget allowance for standard hours 
($3 x 1,000 x 2)........................................................................................          6,000
.................................................................................................................  $        
   300
..............................................................................................unfavorable

The following questions are based on materials in the Appendix to the chapter.

A 38. In the alternate three­variance method, the efficiency variance is:
A. Standard factory overhead rate x (Actual units of allocation base ­ Standard units of allocation 
base allowed)
B. Actual factory overhead incurred ­ Budget allowance based on actual hours
C. Budget allowance based on actual hours ­ (Actual hours x Factory overhead rate)
D. Budgeted fixed factory overhead ­ (Actual hours x Fixed overhead rate)
E. none of the above

D 39. The four­variance method reconciles to the two­variance method by combining which of the following
to get the controllable variance?
A. fixed efficiency variance and idle capacity variance
B. spending variance and fixed efficiency variance
C. spending variance and idle capacity variance
D. spending variance and variable efficiency variance
E. none of the above

B 40. The four­variance method reconciles to the two­variance method by combining which of the following
to get the volume variance?
A. spending variance and variable efficiency variance
B. fixed efficiency variance and idle capacity variance
C. variable efficiency variance and fixed efficiency variance
D. spending variance and idle capacity variance
E. none of the above
Standard Costing:  Setting Standards and Analyzing Variances 130

You might also like